Info

🌱 來自: sxtodx

transient loss of consciousness🚧 施工中

transient loss of consciousness

Scott D. C. Stern, MD

CHIEF COMPLAINT

PATIENT

Mr. M is a 23-year-old medical student who lost consciousness this morning after entering his anatomy lab for the first time. He is quite alarmed (and embarrassed).

What is the differential diagnosis of transient loss of consciousness? How would you frame the differential?

CONSTRUCTING A DIFFERENTIAL DIAGNOSIS

Transient loss of consciousness may be caused by trauma, intoxication, seizures, hypoglycemia, subarachnoid hemorrhage, cerebrovascular disease (involving the brainstem), or syncope. Although syncope is often incorrectly assumed to be synonymous with transient loss of consciousness, syncope actually refers to that subset of patients with transient loss of consciousness due to transient global cerebral hypoperfusion, which in turn is virtually always caused by transient profound hypotension. Therefore, the first pivotal step in the evaluation of patients with transient loss of consciousness is to distinguish syncope from nonsyncopal causes of transient loss of consciousness. Three critical characteristics help distinguish patients with syncope: Syncope is (1) abrupt in onset, (2) brief in duration, and (3) recovery is complete and spontaneous (Figure 31-1). The explanation for this is straightforward; because syncope is due to transient global cerebral hypoperfusion, the loss of consciousness is abrupt. Furthermore, restoration of blood flow must occur quickly or the patient would die rather than present with syncope. Therefore, syncope is brief. Finally, when blood flow is restored, spontaneous recovery occurs promptly. Patients with other features (eg, a prolonged recovery period) should be evaluated for nonsyncopal causes that may masquerade as syncope (eg, seizures, hypoglycemia). A useful question is to ask the patient what is the next thing the patient remembers after losing consciousness. Any significant persistence of confusion beyond a minute or two is critical because this would suggest a nonsyncopal etiology of the transient loss of consciousness (such as a postictal period from a seizure).

Figure 31-1. Identifying syncope

The second pivotal step in patients with syncope identifies the likely category of their syncope: reflex, orthostatic, or cardiac syncope (Figure 31-2). This narrows the differential diagnosis, since each of these categories is associated with specific underlying diseases. Importantly, this key step also helps identify patients with cardiac syncope who are at a substantially increased risk for sudden death. Sudden death may occur if the underlying cardiac process that caused the syncope (arrhythmias or obstruction [eg, aortic stenosis]) is prolonged rather than brief.

Figure 31-2. Distinguishing cardiac, reflex and orthostatic syncope.

Patients with syncope should be carefully evaluated to determine if they have cardiac syncope and are at increased risk for sudden cardiac death.

The evaluation of all syncopal patients must include a thorough history, physical exam, and ECG. A detailed history of the event is critical and includes a description of the setting (prolonged standing, warm environment), prior volume loss (vomiting, diarrhea, melena, rectal bleeding), exactly what the patient was doing (exerting themselves), and the position they were in just prior to their syncopal event (standing, sitting, supine). Additionally, patients should be asked about triggers (pain, anxiety), prodromal symptoms that proceeded the syncope (nausea, abdominal pain), associated symptoms (chest pain, palpitations, shortness of breath), and any signs observed by bystanders. The patient’s medications should be reviewed and their past medical history scrutinized to look for any history of cardiac disease (including ischemic, valvular, or heart failure [HF]). The physical exam should check vital signs and orthostatic BPs in addition to a thorough cardiac and neurologic exam. Finally, an ECG should be obtained in every syncopal patient and examined for signs of arrhythmia, conduction disease, ischemia, or structural heart disease.

Patients with suspected cardiac syncope should be admitted for evaluation (Figure 31-3). Clues that suggest cardiac syncope include a prior history of cardiac disease; syncope while supine, sitting, or during exercise; associated symptoms of chest pain, palpitations, or shortness of breath; a family history of sudden death; or age > 60 years. Physical exam clues include an abnormal rhythm, significant murmur, gallop, jugular venous distention (JVD), lung crackles, or significant edema. Finally, cardiac syncope should be suspected in patients with ECG abnormalities (abnormal rhythm, bundle branch block [BBB], ischemic changes (new or old), left ventricular hypertrophy [LVH], long QT, or preexcitation).

Figure 31-3. Cardiac syncope.

It is useful to appreciate that since there are only 3 categories of syncope, one should also suspect cardiac syncope in patients whose description fits neither orthostatic or reflex syncope.

Orthostatic syncope is suggested by a history of syncope immediately upon standing or pronounced orthostatic hypotension on physical exam whereas syncope with prolonged standing or associated with abdominal discomfort suggests vasovagal syncope.

The last step in patients with orthostatic or cardiac syncope is to identify the specific etiology. Table 31-1 identifies various clues on history, physical exam, ECG, and echocardiography that can suggest specific causes of cardiac syncope.

Table 31-1. Clues to cardiac syncope.

Differential Diagnosis of Transient Loss of Consciousness

A.  Nonsyncopal causes

1.  Generalized seizures

2.  Cerebrovascular disease

a.  Vertebrobasilar insufficiency

b.  Subclavian steal

c.  Subarachnoid hemorrhage

1.  Hypoglycemia

2.  Trauma

3.  Intoxication

B.  Syncope

1.  Reflex syncope

a.  Vasovagal syncope

b.  Situational syncope (cough, micturition, or defecation)

c.  Carotid sinus syndrome

2.  Orthostatic syncope

a.  Dehydration (vomiting, diarrhea, uncontrolled diabetes, overdialysis)

b.  Hemorrhage

c.  Autonomic failure

(1)  Primary autonomic failure: multisystem atrophy, Parkinson disease

(2)  Secondary autonomic failure: diabetes mellitus, vitamin B12 deficiency, uremia

d.  Medications (diuretics, alpha-blockers, vasodilators, nitrates)

3.  Cardiac syncope

a.  Arrhythmias

(1)  Tachycardias

(a)  Ventricular tachycardia (VT)

i.  Secondary to structural heart disease (eg, HF, ischemic heart disease, acute myocardial infarction [MI], valvular heart disease)

ii.  Congenital (eg, long QT syndrome, Brugada syndrome)

iii.  Electrolyte derangements or hypoxia

iv.  Medications (tricyclic antidepressants, antiarrhythmics, phenothiazines, macrolides, protease inhibitors, nonsedating antihistamines, and diuretics [due to electrolyte abnormalities])

(b)  Rapid supraventricular tachycardia (eg, Wolff-Parkinson-White [WPW] syndrome)

(2)  Bradycardias

(a)  Sinus node disorders

i.  Sinus bradycardia (< 35 bpm)

ii.  Sinus pauses (> 3 seconds or > 2 seconds with symptoms)

(b)  Atrioventricular (AV) block (second- or third-degree)

b.  Structural (obstruction)

(1)  Aortic stenosis

(2)  Hypertrophic cardiomyopathy (HCM)

(3)  Pulmonary embolism (PE)

(4)  Rare causes: atrial myxoma, prosthetic valve dysfunction, aortic dissection

Mr. M reports that he was in his usual state of health and felt perfectly well prior to entering the anatomy dissection room. Upon viewing the cadaver, he felt queasy and warm. He became diaphoretic and collapsed to the floor. When he regained consciousness, he was very embarrassed but not confused. The instructor told him that he was unconscious for only a few seconds.

At this point, what is the leading hypothesis, what are the active alternatives, and is there a must not miss diagnosis? Given this differential diagnosis, what tests should be ordered?

RANKING THE DIFFERENTIAL DIAGNOSIS

As noted previously the first step in the evaluation of patients with transient loss of consciousness is to distinguish syncope from other causes. The abrupt onset, brief duration, and spontaneous unaided recovery are entirely consistent with syncope (Figure 31-1). The next step in the evaluation of such patients is to distinguish cardiac syncope from reflex or orthostatic syncope (Figure 31-2). The setting (associated with a strong emotional trigger) and associated prodromal symptoms of nausea and warmth are classic for vasovagal syncope, which is clearly the leading hypothesis. However, it is critical to also consider cardiac syncope, which is potentially life-threatening and is a must not miss hypothesis. While most causes of cardiac syncope are uncommon in young patients, HCM and channelopathies, such as the long QT syndrome, can present in children and young adults and must be considered. Indeed, patients with the long QT syndrome may have life-threatening arrhythmias triggered by emotional stress. Finally, orthostatic syncope is an alternative hypothesis that must also be considered. Table 31-2 lists the differential diagnosis and a careful history, physical exam and review of his ECG are required (Figure 31-2).

Table 31-2. Diagnostic hypotheses for Mr. M.

Mr. M reports no diarrhea, vomiting, melena or rectal bleeding and he is not taking any medications. He has no known heart disease and exercises vigorously without symptoms. He reports no associated chest pain, palpitations, or dyspnea. There is no family history of sudden cardiac death. On physical exam, his BP and pulse are normal and do not change with standing. Cardiac exam reveals a regular rate and rhythm without a significant murmur, JVD, S3 gallop, crackles or edema. His ECG is normal.

Is the clinical information sufficient to make a diagnosis? If not, what other information do you need?

Leading Hypothesis: Reflex Syncope due to Vasovagal Syncope

Textbook Presentation

Vasovagal syncope typically occurs in young patients following a trigger (prolonged standing with or without an emotional stressor) and is preceded by prodromal features (warmth, nausea, diaphoresis, sweating, and lightheadedness).

Disease Highlights

A.  Reflex syncope refers to a group of related disorders that trigger inappropriate cardiovascular reflexes producing hypotension and syncope.

1.  The predominant reflex may be bradycardia (cardioinhibitory type), vasodilatation (vasodepressor type), or both.

2.  This distinction may affect the choice of therapy, with pacemakers a potential option for patients with severe, recurrent symptomatic cardioinhibitory reflex syncope.

3.  Types of reflex syncope include vasovagal (or neurocardiogenic syncope), situational syncope, and carotid hypersensitivity.

4.  The triggers vary with the type of reflex syncope:

a.  Vasovagal syncope: upright posture with or without stress

b.  Carotid sinus hypersensitivity: Carotid pressure, see below

c.  Situational syncope: associated with defecation, micturition, or prolonged coughing

B.  The remainder of this section focuses on vasovagal syncope.

1.  Most common cause of syncope (20–33% of cases)

2.  The pathophysiology is illustrated in Figure 31-4 and includes:

Figure 31-4. Pathophysiology of vasovagal syncope

a.  Prolonged standing causes venous pooling, decreasing venous return thereby decreasing left ventricular (LV) preload (which may be accentuated by dehydration).

b.  Superimposed anxiety, pain, or fear triggers a sympathetic surge, which augments ventricular contraction.

c.  Vigorous contraction coupled with decreased LV preload results in a markedly low end-systolic volume, which triggers intracardiac mechanoreceptors.

d.  The mechanoreceptors trigger the vagal reflex.

e.  The vagal reflex triggers bradycardia, vasodilatation, or both, resulting in hypotension and syncope.

Evidence-Based Diagnosis

A.  History

1.  No single finding is very sensitive for vasovagal syncope (14–60%).

a.  Triggers include prolonged standing (37%), warm environment (42%), lack of food (23%), fear (9–21%), and acute pain (14%).

b.  Prodromal symptoms include sweating (32–66%), nausea (13–60%), and warmth (6–18%).

c.  Venous pooling after exercise may also trigger vasovagal syncope. However, syncope during exercise suggests cardiac syncope.

2.  However, certain findings are fairly specific and increase the likelihood of vasovagal syncope when present.

a.  Prolonged standing (LR+, 9.0)

b.  Abdominal discomfort prior to syncope (LR+, 8)

c.  Occurring during injection/cannulation (LR+, 7)

d.  Dehydration (LR+, 3.7)

e.  Nausea after syncope (LR+, 3.5)

B.  Laboratory and radiologic tests

1.  Patients with a typical history, a normal physical exam and ECG, and no red flags (syncope while supine, sitting or during exertion, associated chest pain, palpitations or dyspnea, age over 60, or prior history of heart disease or family history of sudden death [see Figure 31-2]) do not require further testing.

2.  Patients over 40, those with an atypical history (ie, without a clear precipitant) and those with heart disease or red flags require additional evaluation including an echocardiogram and potentially tilt-table testing and/or an implantable loop recorder (ILR).

3.  Tilt-table testing in vasovagal syncope

a.  The patient is initially supine for 20–45 minutes.

b.  The table is then tilted to 70 degrees and the patient kept upright for 30–40 minutes during which time the pulse and BP are continuously monitored.

c.  Isoproterenol and sublingual nitroglycerin may be given which increase sensitivity but decrease specificity.

d.  Criteria for a positive test include the reproduction of the presyncopal or syncopal symptoms with hypotension, bradycardia, or both.

e.  Test characteristics are estimates due to the lack of a gold standard. However, a recent study utilizing ILRs in patients older than 40 years with presumed recurrent vasovagal syncope revealed the limited accuracy of tilt-table testing.

(1)  No criteria were very sensitive (26–56%).

(2)  Positive tilt-table results are not specific for vasovagal syncope.

(a)  Tilt-table testing can be positive in patients without a history of syncope.

(b)  Tilt-table testing can trigger vagal reflexes and syncope in patients with syncope due to structural heart diseases and arrhythmias, potentially leading to a misdiagnosis of vasovagal syncope.

(c)  Care must be taken prior to assigning the diagnosis of vasovagal syncope to patients with positive tilt-table testing.

Positive tilt-table tests results are not specific for neurocardiogenic syncope.

4.  ILRs

a.  ILRs are devices implanted into the left pectoral region that can record arrhythmias during syncope for up to 36 months.

b.  Useful in patients with recurrent events that may not be captured with short-term continuous loop event monitors and have not been reproduced using tilt table.

c.  In 1 large study of patients with ≥ 3 syncopal episodes believed secondary to vasovagal syncope, the diagnostic yield of ILRs was 37%.

d.  Vasovagal syncope was confirmed in 21–32% but importantly an alternative arrhythmia was diagnosed in 5–16% and 24% of patients were identified who might benefit from a pacemaker.

5.  An approach to reflex syncope is illustrated in Figure 31-5.

Figure 31-5. Diagnostic approach to possible reflex syncope.

Treatment

A.  Patients should be reassured, instructed to avoid triggers, and lie down if they notice the premonitory signs of an impending faint.

B.  Reduction in BP lowering medications (and alcohol) can markedly reduce the incidence of syncope in patients with recurrent vasodepressor vasovagal syncope (NNT 3).

C.  Counterpressure maneuvers (hand grip, arm tensing, squatting, and leg crossing) in which the muscles are tensed for 2 minutes significantly raises BP and can decrease vasovagal syncope (absolute risk reduction 19%, NNT 5).

D.  Midodrine is an alpha-agonist that may be useful. However, compliance is limited due to its 3 times daily dosing requirements and adverse effects (urinary retention).

E.  Fludrocortisone and beta-blockers have not been proven effective.

F.  Pacemakers are useful for select patients with severe recurrent documented cardioinhibitory reflex syncope (pauses > 3 s with symptoms or asymptomatic and > 6 s or AV block) refractory to other treatments (NNT 3).

MAKING A DIAGNOSIS

Mr. M’s well-defined precipitant for vasovagal syncope and typical premonitory symptoms combined with the absence of red flags for serious cardiac syncope (such as HF, ischemic heart disease, advanced age, abnormal physical exam, or ECG) makes neurocardiogenic syncope the most likely diagnosis. His normal ECG rules out the long QT syndrome. He has no history of dehydration and no orthostatic changes on physical exam, effectively ruling out orthostatic hypotension. You still wonder if you need to consider HCM.

Have you crossed a diagnostic threshold for the leading hypothesis, neurocardiogenic syncope? Have you ruled out the active alternatives? Do other tests need to be done to exclude the alternative diagnoses?

Alternative Diagnosis: Hypertrophic Cardiomyopathy (HCM)

Textbook Presentation

HCM may be asymptomatic and discovered due to a family history of sudden cardiac death, during the evaluation of an asymptomatic systolic murmur, during preparticipation athletic screening, or when symptoms occur (syncope, HF, atrial fibrillation, or cardiac arrest).

Disease Highlights

A.  The most common cause of cardiovascular death in young people and among young athletes

B.  A variety of mutations in sarcomere constituents (eg, myosin) result in myocyte hypertrophy with disarray, increased cardiac fibrosis, and diastolic dysfunction. Over 1400 mutations (usually autosomal dominant) in 11 genes have been reported.

C.  Affects 0.02–0.23% of adults in the general population

D.  The hallmark of the disease is LVH in the absence of loading conditions (hypertension, aortic stenosis, etc.)

1.  LVH may develop in childhood, adolescence, or adulthood.

2.  LVH can affect any part of the LV, although often preferentially affects the ventricular septum, which can cause left ventricular outflow tract obstruction (LVOTO).

3.  LVOTO increases the risk of progression to HF, stroke, and sudden cardiac death. The outflow obstruction can be fixed or dynamic.

4.  The pathophysiology of obstruction is complex and due to (1) septal hypertrophy causing narrowing of the LV subaortic outflow tract and (2) the subsequent drag pulling the mitral valve leaflets of into contact with the outflow tract. The motion of the mitral valve may also cause mitral regurgitation.

5.  Chamber size affects the severity of obstruction. A smaller chamber size (ie, from hypovolemia) brings the anterior leaflet of mitral valve closer to the hypertrophied septum and increases obstruction. This occurs when preload decreases (such as with standing), when afterload decreases or when contractility increases.

E.  Most patients are asymptomatic or mildly symptomatic.

F.  Complications include HF, angina, mitral regurgitation, atrial fibrillation, stroke, syncope, and sudden cardiac death.

1.  HF

a.  Typically due to diastolic dysfunction, but systolic dysfunction may develop

b.  More common in patients with LVOTO

c.  Develops due to a combination of outflow obstruction and diastolic dysfunction

d.  Dyspnea on exertion is the most common symptom.

e.  Aggravated by concomitant mitral regurgitation when present

2.  Angina

a.  May be typical or atypical in quality

b.  Develops in 25–30% of patients

c.  May occur secondary to one or more of the following:

(1)  Ischemia without CAD from mismatched supply and demand

(2)  CAD

(3)  LVOTO

3.  Syncope

a.  Develops in 15–25% of patients with HCM

b.  May be due to ventricular arrhythmias, outflow tract obstruction and, rarely, conduction blocks

c.  Unexplained syncope in HCM is a risk factor for sudden cardiac death.

4.  Sudden cardiac death is the most dreaded complication.

a.  Often occurs in previously asymptomatic patients

b.  Usually secondary to ventricular tachyarrhythmias (which may be triggered by myocardial fibrosis and disarray, outflow tract obstruction, or ischemia). Occasionally due to asystole, heart block, pulseless electrical activity or thromboembolism.

c.  Annual risk among all patients with HCM: 1–2%

d.  Major risk factors include the following:

(1)  Prior events

(a)  Prior cardiac arrest

(b)  Spontaneous sustained VT

(2)  High-risk clinical factors

(a)  Family history of sudden cardiac death in first-degree relative younger than 40 years (with or without HCM) or in first-degree relative of any age in whom diagnosis of HCM was established

(b)  Unexplained syncope (particularly if repetitive, exercise-induced, or occurs in children)

(c)  Massive LVH (≥ 30 mm)

(3)  Other risk factors

(a)  Abnormal BP response to exercise (drop > 20 mm Hg) in patients 40 years or younger

(b)  Nonsustained VT on ambulatory cardiac (Holter) monitoring ≥ 3 beats at ≥ 120 bpm

(c)  Young age

(d)  LVOTO

(e)  Increased left atrial size

(f)  Late gadolinium enhancement on cardiac MRI

(4)  Electrophysiologic studies (EPS) are not recommended for routine risk stratification.

5.  Atrial fibrillation

a.  Develops in 20% of patients with HCM

b.  Left atrial enlargement may develop secondary to decreased LV compliance or mitral regurgitation and creates a substrate for the development of atrial fibrillation.

c.  Atrial fibrillation decreases LV filling and worsens the outflow tract obstruction.

d.  Atrial fibrillation markedly increases the risk of thromboembolism (OR 17.7 compared with patients in sinus rhythm).

6.  Stroke is usually secondary to concomitant atrial fibrillation and subsequent embolization.

Evidence-Based Diagnosis

A.  Echocardiogram is a typical diagnostic test of choice.

1.  Criteria for HCM include LV wall thickening (≥ 15 mm) in the absence of other conditions known to cause LVH (ie, hypertension or aortic stenosis).

2.  LVH can occur in any part of the LV and in an array of distributions but is often asymmetric in distribution.

3.  The classic pattern that has specific consequences is marked by septal hypertrophy.

B.  Cardiac MRI and transesophageal echocardiography can be used when transthoracic echocardiography is suboptimal. Late enhancement with gadolinium identifies myocardial fibrosis and is associated with an increased risk of sudden cardiac death.

C.  Physical exam

1.  Patients with HCM may have (1) a normal physical exam or (2) the systolic murmur caused by LVOTO, a harsh systolic murmur at the apex and lower left sternal border and/or (3) the murmur of mitral regurgitation due to systolic anterior motion of the mitral valve leading to poor valve coaptation and mitral regurgitation

2.  The classic HCM murmur is caused by LVOTO and accentuated by maneuvers that decrease chamber size (causing increased obstruction).

a.  The murmur increases as a patient goes from a squatting to a standing position (sensitivity, 95%; specificity, 84%; LR+, 5.9; LR−, 0.06).

b.  Passive leg elevation decreases the murmur (sensitivity, 85%; specificity, 91%; LR+, 9.4; LR−, 0.16).

D.  ECG findings

1.  ECG abnormalities may precede echocardiographic abnormalities and become more frequent with age.

2.  Abnormalities include repolarization changes (ST-segment elevation, depression or T wave inversions) and voltage criteria for LVH. Other findings may include prominent Q waves, left atrial enlargement, and left axis deviation.

3.  Abnormal in 86–90% of affected patients with echocardiographic LVH and 46% of affected patients (gene positive) without LVH.

4.  Voltage criteria for LVH are present in 65% of patients with echocardiographic LVH and 32% of gene-positive patients without echocardiographic LVH.

E.  DNA analysis

1.  DNA analysis can identify mutant genes but remains imprecise.

2.  Less than 50% of affected patients have an identifiable mutation and many mutations are of uncertain significance.

3.  Testing relatives of patients with HCM can be useful, especially when probands have a known pathogenic mutation. Testing can both identify affected relatives before the development of LVH and can also rule out the disease.

F.  Distinguishing HCM from athlete’s heart

1.  Highly trained athletes occasionally have significant LVH (13–15 mm), which can suggest HCM. A variety of features can be useful to make the distinction in such patients.

2.  Features that suggest HCM include

a.  A smaller LV end-diastolic volume < 55 mm (100% sensitive, 100% specific)

b.  T wave inversions (52% sensitive, 93% specific; LR+, 7.4; LR–, 0.52)

c.  A family history of HCM

d.  Late gadolinium enhancement on cardiac MRI

Treatment

A.  Evaluation

1.  Initial and annual laboratory tests should include a 12-lead ECG, transthoracic echocardiography, 48-hour ambulatory cardiac (Holter) monitor and continuous loop event monitor (for patients with palpitations or light headedness), and a symptom limited stress test (to assess BP response to exercise and evaluate ischemia).

2.  Cardiac MRI with late gadolinium enhancement should be considered to further delineate anatomy.

3.  LVOTO should be evaluated.

a.  LVOTO contributes to the risk of sudden death and is considered significant if > 50 mm Hg.

b.  Since LVOTO is dynamic, a variety of measures are used, including standing echocardiography (to decrease preload and increase obstruction) and exercise transthoracic echocardiography in symptomatic patients if resting dynamic outflow obstruction is < 50 mm Hg.

B.  Therapy

1.  Asymptomatic patients

a.  Patients should avoid dehydration and strenuous exertion. Low-intensity aerobic exercise is reasonable.

b.  Beta-blockers and calcium channel blockers are unproven in asymptomatic HCM with or without obstruction.

c.  Vasodilators and high-dose diuretics should be avoided in patients with HCM and obstruction.

d.  Septal reduction should not be performed in asymptomatic patients.

2.  Symptomatic patients

a.  LVOTO

(1)  Beta-blockers decrease contractility and slow heart rate, augmenting diastolic filling and thereby decreasing dynamic outflow obstruction. Also recommended in patients with dyspnea or angina

(2)  Disopyramide can be used if beta-blockers are ineffective.

(3)  Verapamil or diltiazem

(a)  Can be used if beta-blockers are ineffective or not tolerated

(b)  Verapamil should not be used concurrently with beta-blockers due to a high frequency of heart block and HF

(c)  Other contraindications include advanced HF, high gradients, or sinus bradycardia.

(4)  Patients with symptomatic (but not asymptomatic) LVOTO with gradients ≥ 50 mm Hg

(a)  Symptoms may include HF despite maximum medical therapy, or recurrent exertional syncope due to LVOTO

(b)  Should be considered for septal ablation to decrease the obstruction

(c)  Options include surgical septal myectomy or intracoronary alcohol infusion into septal perforators, which induces necrosis.

(5)  Patients should avoid dehydration; excess alcohol consumption; and a variety of drugs, including digoxin, positive inotropes, arterial and venodilators (including dihydropyridines, nitrates, and the phosphodiesterase inhibitors used for erectile dysfunction [sildenafil, tadalafil, and vardenafil])

b.  Chest pain evaluation

(1)  Patients with HCM have a high frequency of nonspecific ECG and nuclear imaging abnormalities, limiting the utility of stress testing.

(2)  Coronary angiography is recommended in patients with typical exertional angina, survivors of sudden death, and patients with sustained VT.

c.  Syncope

(1)  The recommended evaluation for patients with HCM and syncope include a12-lead ECG, exercise stress echocardiography, and 48-hour ambulatory cardiac (Holter) monitoring.

(2)  The risk of sudden cardiac death should be evaluated. Those at high risk should receive an implantable cardiac defibrillator (ICD). Those at low risk could be further evaluated with ILR.

d.  HF

(1)  With LVOTO (see above)

(2)  HFrEF < 50%:

(a)  Angiotensin-converting enzyme (ACE) inhibitor (or angiotensin receptor blockers) and beta-blockers are recommended.

(b)  Low-dose diuretics can be used if dyspnea persists despite treatment with beta-blockers and ACE inhibitor. Mineralocorticoid receptor antagonists should be considered.

(3)  Heart failure with preserved ejection fraction:

(a)  Verapamil or diltiazem should be considered if beta-blockers are ineffective.

(b)  Low-dose diuretics should be considered if symptoms persist.

e.  Atrial fibrillation: Due to the high risk of thromboembolism in patients with HCM and atrial fibrillation, anticoagulation is recommended.

C.  Implantable cardiac defibrillator (ICD)

1.  ICDs are the most effective strategy to prevent sudden cardiac death in patients with HCM.

2.  Recommended for high-risk HCM patients.

a.  ICD therapy is recommended for HCM patients with a history of a prior cardiac arrest or hemodynamically significant spontaneous sustained VT.

b.  ICD implantation is reasonable in HCM patients at increased risk for sudden cardiac death due to a family history of sudden cardiac death in first-degree relative, recent unexplained syncope or massive LVH, and select patients with nonsustained VT (≥ 120 bpm ≥ 3 consecutive beats) or abnormal BP response to exercise in the presence of other risk factors.

3.  A calculator has been developed by the European Society of Cardiology to estimate risk. However, 1 analysis of a large number of patients, suggested that the calculator was not sufficiently sensitive to identify patients at risk for sudden cardiac death. The “high risk” score was only 20% sensitive for sudden cardiac death, 93% specific, LR+ 2.6, LR– 0.9. Even a moderate risk score was insufficiently sensitive (41%, LR– 0.8)

4.  Patient selection is complex and recommendations are in evolution. Consultation is advised.

D.  Screening

1.  First-degree relatives of affected patients should be referred for genetic counseling to discuss screening for HCM.

2.  In the general population, preparticipation screening with a history (including family history), physical exam, and ECG of all young competitive athletes has been demonstrated to reduce the incidence of sudden cardiac death by 79% primarily due to a reduction in deaths from HCM.

CASE RESOLUTION

As noted above, Mr. M’s history and physical exam and normal ECG suggest vasovagal syncope. There is no family history of sudden cardiac death, significant murmur, or ECG abnormality to suggest either the long QT syndrome or HCM. There is no history of dehydration or offending medications (eg, vasodilators) nor orthostatic changes on exam to suggest orthostatic hypotension. Tilt-table testing is not indicated in patients with isolated episodes of well-defined vasovagal syncope.

Mr. M is reassured, and although embarrassed, he feels much better. After explaining the pathophysiology of his disorder, you initiate standard recommendations for the prevention of further episodes.

CHIEF COMPLAINT

PATIENT

Mr. C is a 65-year-old man with diabetes who comes to see you with a chief complaint of losing consciousness. He reports that he was sitting at home watching television when he suddenly lost consciousness without any warning. His wife reports that he was unresponsive for approximately 30 seconds. There was no tonic-clonic activity or incontinence, and the patient was not confused after regaining consciousness. The patient’s wife reports that she took Mr. C’s blood glucose when he passed out and that the reading was 120 mg/dL.

At this point, what is the leading hypothesis, what are the active alternatives, and is there a must not miss diagnosis? Given this differential diagnosis, what tests should be ordered?

RANKING THE DIFFERENTIAL DIAGNOSIS

As illustrated in Figure 31-1, the first step in patients with loss of consciousness is to determine whether this was due to syncope, or some nonsyncopal cause (eg, trauma, intoxication, hypoglycemia, or seizure). As discussed previously, 3 questions help make this pivotal distinction: Was the loss of consciousness abrupt in onset, brief in duration, and was there spontaneous unaided recovery? Clearly, the story suggests that all 3 were true and he therefore suffered from syncope. (Although hypoglycemia should be considered in any patient with treated diabetes, his recovery without intervention essentially rules that out, as of course, does the normal blood glucose at the time.) The second pivotal step in syncopal patients is to determine whether the patient has cardiac, orthostatic, or reflex syncope. As illustrated in Figure 31-2, this process is driven by a search for risk factors and associated symptoms to determine whether the patient is at risk for cardiac syncope, which could be life-threatening. In particular, patients should be asked about a prior history of heart disease or a family history of sudden death; syncope occurring during exertion or in the supine or seated position; and any associated chest pain, dyspnea, or palpitations.

Mr. C denies any history of exertion prior to his loss of consciousness. He denies any associated chest pain, palpitations, or dyspnea. Past medical history reveals that Mr. C has suffered 2 MIs. Subsequently, he has dyspnea upon walking more than 20 yards. His medications include atenolol, aspirin, atorvastatin, insulin, and lisinopril. On physical exam, his BP is 128/70 mm Hg with a pulse of 72 bpm, which is regular. There is no significant change upon standing. His lung exam is clear, and cardiac exam reveals prominent JVD and a loud S3 gallop. There is no significant murmur. He has 2+ pretibial edema, and his rectal exam reveals guaiac-negative stool.

Is the clinical information sufficient to make a diagnosis? If not, what other information do you need?

Mr. C’s does not have exertional syncope, chest pain, palpitations, or dyspnea at the time of syncope. Nonetheless, his prior history of heart disease substantially increases the likelihood of some form of cardiac syncope, which becomes both the leading and must not miss hypothesis. The likelihood of cardiac syncope is increased even further because Mr. C’s history and exam are not suggestive of the other 2 types of syncope: orthostatic or reflex. (Orthostatic syncope is unlikely given his seated position, the absence of any history of volume loss or bleeding, and more importantly, by the lack of postural BP change with standing. Reflex syncope [due to neurocardiogenic syncope] is also unlikely given the lack of any trigger, prodrome and because Mr. C.’s syncope occurred while he was sitting.) He should be admitted for evaluation while his history, physical exam, and ECG are examined for clues that might suggest the etiology of his probable cardiac syncope (Table 31-1). Prior to ranking the differential, it is useful to review the syndrome of cardiac syncope.

Leading Hypothesis: Cardiac Syncope

Textbook Presentation

Cardiac syncope refers to syncope secondary to disorders arising within the heart. Arrhythmias (either tachyarrhythmias or bradyarrhythmias) are the most common. Less common disorders include acute coronary syndromes, obstructive syndromes (aortic stenosis, HCM and pulmonary embolism), and rare causes (aortic dissection and atrial myxoma). Classically, patients with cardiac syncope are elderly patients with known heart disease (ie, HF or CAD) who experience sudden syncope, which may occur without warning. Patients may have palpitations.

Disease Highlights

A.  Cardiac syncope is associated with markedly increased mortality compared with reflex syncope, orthostatic syncope, or syncope of unknown cause.

B.  Among patients with dilated cardiomyopathy, sudden cardiac death (presumably arrhythmogenic) accounts for 30% of the mortality.

C.  Patients in whom cardiac syncope is suspected should be admitted for evaluation.

D.  Although there are a large number of cardiac dysrhythmias, only a relative few produce syncope. Most supraventricular tachyarrhythmias will not cause syncope because the AV node limits the ventricular response rate. The most common arrhythmias associated with syncope include

1.  Tachycardias

a.  VT

b.  Supraventricular tachycardias associated with an accessory pathway (ie, WPW syndrome) or those associated with a very rapid ventricular responses.

2.  Bradycardias

a.  Sinus node dysfunction

(1)  Sinus bradycardia (< 35 bpm)

(2)  Sinus pauses (defined as > 3 seconds or > 2 seconds with symptoms)

b.  AV heart block (second- or third-degree)

c.  Atrial fibrillation with a slow ventricular response

Evidence-Based Diagnosis

A.  History

1.  Certain clinical findings substantially increase the likelihood of cardiac syncope when present, including

a.  Preexistent heart disease

b.  An abnormal ECG

c.  Syncope associated with chest pain

d.  Syncope during exertion (LR+, 6.5–14)

e.  Syncope when supine (LR+, 2.5 –∞) or sitting

Syncope during exertion is unusual but worrisome and suggests cardiac syncope.

2.  Other symptoms may suggest cardiac syncope but are less specific (palpitations, syncope of sudden onset, dyspnea associated with syncope).

3.  A careful consideration of a patient’s associated symptoms, physical exam findings, or ECG abnormalities may suggest a particular cause of cardiac syncope (see Table 31-1).

4.  Cardiac syncope is unlikely in patients without known or suspected cardiac disease on the basis of the initial history, physical exam, and ECG (LR–, 0.09–0.12).

5.  Table 31-3 summarizes the sensitivity, specificity, and LR for symptoms in predicting cardiac syncope.

Table 31-3. Sensitivity, specificity, and LRs for cardiac syncope.

B.  ECG

1.  An abnormal ECG increases the OR of cardiac arrhythmias in patients without vasovagal syncope (OR, 23.5 [CI, 7 – 87]).

2.  Certain ECG findings in patients with syncope may suggest particular cardiac etiologies (Table 31-1).

a.  ECG evidence of prior MI or a long QT interval increases the likelihood of VT.

b.  ECG findings of significant bradycardia, second- or third-degree AV block increase the likelihood of syncope due to sick sinus syndrome (SSS) or AV block.

c.  Bundle branch block (BBB) on ECG increases the likelihood of both AV block and VT.

(1)  Mortality in patients with syncope and BBB is 28% at 40 months. 32% of the deaths were sudden death.

(2)  The increased mortality is attributed to a combination of VT or electromechanical dissociation in patients with underlying heart disease.

(3)  AV block develops in 17% of patients with BBB and syncope.

(4)  EPS can document prolonged conduction through the His-Purkinje system and is highly specific but insensitive (67%). ILR recordings can also be useful.

(5)  Pacemaker therapy effectively prevents syncope in almost all such patients (but does not prevent sudden death).

d.  Right ventricular strain (S1Q3T3) or right BBB suggests PE.

e.  Ischemic changes suggest MI.

f.  Delta wave or short PR interval suggests an accessory pathway (eg, WPW syndrome).

C.  Clinical decision rules have been developed that combine clinical risk factors and results from the ECG to predict cardiac syncope and mortality. However, clinical judgment was more sensitive at predicting serious 30-day events than any of the scoring systems (94% vs. ≈ 75%).

D.  Other tests

1.  Echocardiograms

a.  Useful in patients with suspected aortic stenosis and HCM (ie, systolic murmur)

b.  Useful in patients with possible cardiac syncope (a prior history of cardiac disease; a family history of sudden cardiac death; syncope while supine, sitting, or with exertion; associated with chest pain, palpitations, or dyspnea; an abnormal cardiac exam; or an abnormal ECG). Significant abnormalities (ejection fraction ≤ 40%) found in 27% of such patients and 50% of these patients diagnosed with significant arrhythmias.

c.  Not useful in patients with unexplained syncope without either a cardiac history or an abnormal ECG.

d.  Table 31-1 summarizes echocardiographic clues in patients with syncope.

2.  B-type natriuretic peptide and troponin

a.  Neither has demonstrated sufficient accuracy for the diagnosis of cardiac syncope

b.  The 2017 ACC/AHA/HRS Syncope Guideline only recommended their use if MI or HF is suspected

3.  Exercise testing

a.  Particularly useful in patients with exertional syncope, chest pain, or ischemic ECG changes

b.  Should be performed with extreme caution

c.  May be useful in patients with dyspnea on exertion

4.  Cardiac monitoring:

a.  Diagnostic only if

(1)  Arrhythmia captured and patient symptomatic during arrhythmia or

(2)  Rhythm normal during symptoms (excludes an arrhythmia)

(3)  The European Society of Cardiology Guidelines also considers certain significant arrhythmias to be diagnostic even if asymptomatic including Mobitz II second- or third-degree AV block, rapid supraventricular tachycardia > 160 bmp for > 32 beats, VT or asystolic pauses ≥ 3 sec (except young trained persons or during sleep or rate-controlled atrial fibrillation).

b.  A variety of devices can monitor the patient’s rhythm and transmit the recordings automatically or when triggered.

c.  Devices vary in their duration of monitoring (days for ambulatory cardiac [Holter] monitoring to weeks for event recorders to years for ILRs).

d.  ILRs have been used successfully in some patients with recurrent unexplained syncope. Devices can stay in place up to 2–3 years. The yield in such patients is often higher (55%) than with external monitors (19%). Importantly, 19% of such patients had severe symptomatic bradyarrhythmias (usually asystole). Another 11% of patients had no arrhythmia during syncope, ruling out an arrhythmogenic cause.

e.  Device selection depends on the frequency of symptoms (longer duration important for patients with infrequent symptoms) and the ability of patients to trigger the device

5.  EPS require a right heart catheterization. During EPS, stimuli are delivered in order to elicit tachyarrhythmias and detect accessory pathways. Bradyarrhythmias may be implied when patients have prolonged conduction times or when the sinus node responses to rapid pacing are abnormal.

a.  Sensitivity is 90% for VT.

b.  Sensitivity for bradyarrhythmias is low (33%).

c.  Overall diagnostic yield of EPS 50% in patients with heart disease and 10% in patients without heart disease, and 22% in patients with an abnormal ECG vs. 3.7% in patients with a normal ECG.

d.  Indications for EPS in patients with unexplained syncope include

(1)  Prior MI

(2)  Structural heart disease

(3)  Impaired LV function

(4)  Bifascicular block

(5)  Monitoring suggests sinus node dysfunction or AV block

e.  EPS unnecessary for patients with a class I indication for an implantable cardiac defibrillator (ejection fraction ≤ 35%)

f.  The European Society of Cardiology concluded EPS was not useful in patients with unexplained syncope who did not have structural heart disease, an abnormal ECG, or palpitations.

g.  Risk of EPS include cardiac perforation, MI, AV fistulae, deep venous thrombosis, and PE.

E.  Table 31-1 summarizes the clues on history, physical exam, ECG, and echocardiogram that suggest various causes of cardiac syncope.

In review, Mr. C is a 65-year-old man with a history of CAD, 2 MIs, and recent syncope while sitting.

Is the clinical information sufficient to make a diagnosis? If not, what other information do you need?

Reviewing Table 31-1, suggests that Mr. C’s past history of CAD increases his risk for cardiac syncope due to AV block or VT, both of which are must not miss hypotheses. Furthermore, his history of dyspnea on minimal exertion, JVD, and S3 gallop all suggest HF. HFrEF also markedly increases the likelihood of VT, making this the leading hypothesis. Additionally, given Mr. C’s past medical history of CAD, an acute coronary syndrome, must also be considered (although an uncommon cause of syncope). Finally, another “must not miss” cause of cardiac syncope is PE. Table 31-4 lists the differential diagnosis.

The ECG shows Q waves in leads V1–V4 and II, III, and aVF consistent with prior anterior and inferior MI. These were present on his prior ECG 6 months previously. The PR interval is normal. There is no evidence of sinus bradycardia, sinus pause, or AV block. The QRS width is normal, excluding BBB. An echocardiogram reveals LV dysfunction with hypokinesis of the anterior and inferior walls. The ejection fraction is estimated to be 38%. The aortic valve is normal without evidence of aortic stenosis.

Reviewing the differential in Table 31-4, Mr. C does not have acute chest pain or ECG changes to suggest a new acute coronary event. The absence of BBB or AV block on the current ECG decreases the likelihood of AV block but does not exclude it, since AV block can be intermittent. His echocardiogram confirms HFrEF, which increases his risk for VT.

Table 31-4. Diagnostic hypotheses for Mr. C.

Revised Leading Hypothesis: VT

Textbook Presentation

Patients with VT may be asymptomatic or have symptoms that range from palpitations to light-headedness, near syncope, syncope, or sudden cardiac death.

VT occurs most commonly in patients with heart disease and should be seriously considered in patients with syncope and a history of preexisting CAD, HF, or other heart disease.

Disease Highlights

A.  Etiology and associations

1.  Ischemic heart disease

a.  Associated with CAD in 80% of cases

b.  May be secondary to acute ischemia/MI or prior scar

c.  VT and ventricular fibrillation complicates 10% of STEMIs (ST-segment elevated MI)

2.  HF

3.  Other heart diseases: HCM, valvular heart disease, infiltrative disorders

4.  Miscellaneous causes

a.  Electrolyte disorders (hypokalemia and hypomagnesemia)

b.  Hypoxia

c.  Drugs, particularly those that prolong the QT interval (eg, antiarrhythmics, antipsychotics, tricyclic antidepressants, macrolides, some fluoroquinolones, and many others)

5.  Congenital disorders

a.  Congenital heart disease

(1)  Long QT syndrome

(a)  The ECG of affected families demonstrates long refractory periods (long QT intervals defined as a QTc of > 450 ms in males and 460 ms in females)

(b)  Affected patients are at increased risk for sudden cardiac death from a form of VT called torsades de pointes.

(c)  Arrhythmias may be precipitated by emotional stress, exercise, loud abrupt noises, or during sleep.

(d)  Several symptoms typically associated with vasovagal syncope are also common in the long QT syndrome: triggered by emotional stress, pain, or noise (70%); sweating (67%); nausea (29%); situational (associated with micturition, defecation, or coughing) (17%); abdominal discomfort (16%).

Long QT syndrome may mimic vasovagal syncope. Even patients with symptoms typical of vasovagal syncope should have an ECG performed and QTc measured.

(e)  Associated with congenital neural deafness

(2)  Brugada syndrome

(a)  Caused by a mutation in the sodium channel gene. Affected patients are predisposed to polymorphic VT and sudden death.

(b)  Suggestive baseline ECG abnormalities include a right BBB pattern with ST elevation in the right precordial leads.

B.  Prognosis

1.  VT is a potentially life-threatening arrhythmia.

2.  Predictors of mortality in patients with VT include prior cardiac arrest, LV dysfunction, post-MI, or inducible VT on EPS.

Evidence-Based Diagnosis

A.  ECG criteria for VT

1.  ≥ 3 consecutive wide complex (QRS ≥ 0.12 seconds) beats (Figure 31-6) > 100 bpm constitutes a wide complex tachycardia (but not necessarily VT)

Figure 31-6. Ventricular tachycardia.

a.  80–90% of wide complex tachycardias are due to VT

b.  However, supraventricular tachycardias also occasionally have wide QRS complexes (> 0.12 s) due to either a concomitant BBB or an accessory pathway, hyperkalemia, or drug-induced QRS changes (tricyclic antidepressants overdose and class 1a antiarrhythmics)

c.  A history of CAD or HF increases the likelihood that the wide complex tachycardia is VT.

d.  ECG criteria that increase the likelihood that the wide complex tachycardia is due to VT rather than supraventricular tachycardia with aberrancy include:

(1)  Capture beats (a narrow supraventricular complex present in the midst of the wide complex tachycardia)

(2)  Fusion beats (a QRS complex with a hybrid morphology between that of the normal narrow QRS complex and that of the wide complex tachycardia). This suggests a supraventricular impulse partially captured the ventricle.

(3)  AV dissociation

(4)  Concordance of the precordial leads (chest leads either entirely all positive or entirely all negative) (Figure 31-7)

Figure 31-7. Positive concordance in VT (Reproduced with permission from ECG Library at https://litfl.com).

(5)  QRS duration > 160 msec, LR+ 22.9

(6)  The absence of these findings does not establish supraventricular tachycardia.

e.  Review of prior ECGs can be helpful.

(1)  A preexistent BBB on a prior ECG (during sinus rhythm) with the same QRS morphology as that during the arrhythmia favors supraventricular tachycardia with aberrancy.

(2)  Evidence of WPW syndrome suggests supraventricular tachycardia with antidromic conduction down the accessory tract.

f.  Hemodynamic stability does not rule out VT.

All wide complex tachycardias should be assumed to be VT unless there is conclusive evidence of a supraventricular tachycardia.

2.  Sustained VT is defined as VT lasting > 30 seconds.

B.  EPS can be performed in patients with risk factors for VT (eg, ischemic heart disease, HF, HCM), but without documented VT.

1.  The ability to provoke sustained monomorphic VT is diagnostic.

2.  This is particularly useful in patients in whom VT is suspected but do not already have an indication for an ICD.

Treatment

A.  Patients with ongoing VT

1.  Unstable patients (with hypotension, angina, HF, or altered mental status) should be managed according to ACLS guidelines.

2.  The evaluation should include

a.  Obtain baseline ECG (if patient sufficiently stable)

b.  Measure troponin level to look for evidence of ischemia.

c.  Check electrolytes (especially potassium, magnesium) and SaO2.

d.  Review medications to search for drugs associated with QT prolongation.

e.  Measure drug levels as appropriate (eg, digoxin)

B.  Follow-up evaluation is directed at evaluating the etiologies of VT and risk for sudden death.

1.  Stress testing (and coronary angiography in selected patients) can help uncover underlying ischemia precipitating VT and is recommended for patients with exercise-induced syncope or chest pain or an intermediate or greater probability of CAD.

2.  An echocardiogram should be obtained to evaluate LV function and rule out valvular heart disease and HF.

3.  EPS are recommended for selected patients.

C.  Prevention of recurrent VT and sudden cardiac death

1.  Treat underlying conditions

a.  Treat ischemic heart disease (including revascularization if necessary)

b.  Treat HF (angiotensin-converting enzyme [ACE] inhibitors, beta-blockade, and spironolactone have all been shown to decrease mortality).

c.  Optimize electrolytes, including magnesium.

2.  Specific therapy for the treatment and prevention of VT includes antiarrhythmic drugs (especially beta-blockers and possibly amiodarone), catheter ablation, ICDs, and combinations of the above.

a.  ICDs are implanted devices that monitor the cardiac rhythm and automatically detect and cardiovert patients in VT.

b.  ICDs are used in selected patients at high risk for sudden death, including select patients with HFrEF, survivors of sudden death, patients with sustained VT and patients in whom syncope was believed to have been caused by VT.

c.  Appropriate use criteria for ICDs have been published by the ACCF/AHA.

Alternative Diagnosis: Acute Coronary Syndrome & Syncope

Acute coronary syndrome is an unusual cause of syncope and is covered extensively in Chapter 9, Chest Pain. This discussion will focus on patients who experience syncope due to an acute coronary syndrome. Briefly, acute coronary syndromes account for approximately 3% of patients presenting to emergency departments with syncope. The mechanism of syncope varies and includes reflex syncope (particularly in inferior MIs), advanced AV block (particularly in anterior MIs) and VT. Symptoms are often atypical in syncopal patients with acute MI; chest pain is present in only 17%, dyspnea in 30%, and a history of CAD in 54%. Lab abnormalities include troponin elevation on presentation (50%) and ST-segment elevation (9%). Nonetheless, a normal ECG (defined as normal sinus rhythm without new or indeterminate changes) makes the diagnosis of acute MI unlikely, negative predictive value 99% (sensitivity, 80%; specificity, 64%; LR+, 2.2; LR–, 0.31).

MAKING A DIAGNOSIS

Mr. C’s serum troponin levels are repeatedly undetectable (thus excluding acute MI.) The pretest probability of VT is very high.

You still wonder if a significant bradyarrhythmia or a PE might be responsible for Mr. C’s syncope.

Have you crossed a diagnostic threshold for the leading hypothesis, VT? Do other tests need to be done to exclude the alternative diagnoses?

Alternative Diagnosis: Bradycardia from SSS

Textbook Presentation

The presentation of SSS depends on the duration and severity of the bradyarrhythmia. When the bradyarrhythmia is severe and prolonged, patients may experience sudden syncope. With less severe bradycardia, patients may experience weakness, dyspnea on exertion, HF, angina, transient ischemic attacks, or near syncope. Since the bradyarrhythmia may be short-lived, patients may recover without intervention.

Disease Highlights

A.  Episodic or persistent failure of sinus node to meet the physiological demands of the patient

B.  Often progressive over time

C.  Electrical manifestations include:

1.  Sinus bradycardia < 40 bpm

2.  Sinus pauses > 2 seconds

3.  Sinus arrest (with an escape junctional rhythm)

4.  Sinoatrial exit block (inability of the sinus impulse to exit the sinus node)

5.  Chronotropic incompetence: In this condition, the sinus rate does not increase appropriately with physical activity, leading to relative bradycardia and symptoms.

6.  Tachy-brady syndrome: SSS is associated with supraventricular tachyarrhythmias, in 40–60% of patients, particularly atrial fibrillation. Such patients may complain of palpitations. The bradycardia often follows termination of the tachycardia. Tachy-brady syndrome markedly increases the risk of death or nonfatal stroke (2- to 3-fold) compared with SSS alone.

D.  Most common indication for pacemaker placement (accounts for 30–50% of all pacemakers)

E.  Age is the leading risk factor (RR 1.73 per 5 y). The mean age of onset is 68 years.

F.  Usually due to fibrosis and degeneration of sinus node.

G.  Underlying CAD is common and contributes to the pathogenesis of SSS in some patients.

H.  A variety of medications can depress sinus node function and aggravate SSS, including beta-blockers, verapamil, diltiazem, digoxin, acetylcholinesterase inhibitors, cimetidine, clonidine, lithium, methyldopa, and other antiarrhythmics.

I.  Less common causes include hypothyroidism, sarcoidosis, amyloidosis, hemochromatosis, pericarditis, Lyme disease, Chagas disease, and rheumatic fever.

J.  Concomitant AV conduction disturbances are present in over 50% of patients with SSS.

Evidence-Based Diagnosis

A.  Simultaneous symptoms and ECG findings (sinus bradycardia, significant pauses or sinus exit block) establishes the diagnosis.

B.  Ambulatory cardiac (Holter) monitoring may be used but is often nondiagnostic due to the intermittent nature of the arrhythmia.

C.  External cardiac continuous loop event monitors allow for a longer period of monitoring and correlation with symptoms.

D.  ILRs have also been used to provide even longer periods of monitoring.

E.  EPS: Occasionally used in patients with severe symptoms when monitoring has failed to capture patients during symptoms (which could confirm or exclude SSS). However, sensitivity is imperfect and normal results do not rule out SSS.

F.  Exercise stress testing can be useful to identify ischemia or chronotropic incompetence.

Treatment

A.  Unstable patients: Treat according to ACLS guidelines

B.  In stable patients, discontinue any medications that may adversely affect sinus function (see above). (If beta-blockers or other drugs cannot be discontinued, patients may require a pacemaker.)

C.  Indications for pacemaker placement

1.  Documented symptomatic sinus node dysfunction (bradycardia or pauses), including those resulting from medications that cannot be discontinued

2.  Symptomatic chronotropic incompetence (inability to obtain an adequate heart rate for exertion)

3.  Pacemakers are reasonable in patients with unexplained syncope when SSS is suspected but cannot be confirmed.

a.  Patients with HR < 40 bpm while awake

b.  EPS reveals evidence of significant sinus node dysfunction.

Alternative Diagnosis: Bradycardia due to AV Heart Block

Textbook Presentation

Depending on the duration and severity of the heart block, patients with AV block may be asymptomatic or complain of syncope, near syncope, sudden cardiac death, palpitations, angina, or transient ischemic attacks.

Disease Highlights

A.  Secondary to conduction abnormalities in the AV node, bundle of His, or bundle branches impairing transmission from atria to the ventricles.

B.  The disturbance may be intermittent or permanent.

C.  Classification (Table 31-5)

Table 31-5. Classification of heart block.

1.  In first-degree AV block, all of the sinus impulses (P waves) are conducted but the PR interval is prolonged.

2.  In second-degree block, some of the impulses are conducted. There are 2 subtypes: Mobitz type I and Mobitz type II (Table 31-5).

3.  In third-degree AV block, none of the P waves are conducted (Figure 31-8).

Figure 31-8. Third-degree atrioventricular block.

4.  In second- or third-degree AV block, the ventricular rate slows and may depend on lower intrinsic pacemakers residing within the ventricle. The bradycardia can cause dyspnea, angina, hypotension, syncope, or death.

D.  AV nodal disease should also be suspected in patients with atrial fibrillation who have a slow ventricular response and are not taking medications that slow AV conduction (eg, digoxin, beta-blockers, verapamil, or diltiazem).

E.  Etiology

1.  Fibrosis of the conduction system

2.  Ischemic heart disease

3.  Medications (eg, beta-blockers, verapamil, diltiazem, digoxin, adenosine, amiodarone)

Most patients with AV block attributed to verapamil, diltiazem, or beta-blockers also have conduction disease and are likely (> 80%) to experience AV block even off medications.

4.  Sarcoidosis is a common cause of unexplained second- or third-degree AV block in patients younger than 60 years (34%) and should be considered even if there is no prior diagnosis of sarcoidosis. 27% of such patients subsequently suffer from VT of HF.

5.  Hyperkalemia

6.  Valvular heart disease (due to extension of calcification into the conduction system)

7.  Increased vagal tone

8.  Miscellaneous other causes (hypothyroidism, Lyme disease, amyloidosis, etc)

9.  Cardiac procedures (ie, transcathetic aortic valve implantation)

Evidence-Based Diagnosis

A.  Diagnosed when any of the following captured on monitoring:

1.  Third-degree AV block

2.  Advanced second-degree AV block (defined as 2 consecutively blocked P waves) with any of the following:

a.  Symptoms

b.  Ventricular arrhythmias presumed secondary to AV block

c.  Asystole ≥ 3 seconds or escape rate < 40 bpm or escape rhythm below AV node

d.  Associated with atrial fibrillation and pauses ≥ 5 seconds

3.  Symptomatic second-degree AV block (regardless of type)

4.  Unexplained syncope with chronic bifascicular block

B.  Long-term monitoring with ILR increases the diagnostic yield.

Treatment

A.  Discontinue medications that impair AV conduction.

B.  Treat ischemia.

C.  Correct electrolyte abnormalities.

D.  Atropine can be useful in emergent situations.

E.  Pacemakers

1.  Precise indications are complex. The ACCF/AHA published guidelines in 2012. Some of the more common indications for pacing include:

a.  Third-degree AV block

b.  Advanced second-degree AV block (defined as 2 consecutively blocked P waves) with any of the following:

(1)  Symptoms

(2)  Ventricular arrhythmias presumed secondary to AV block

(3)  Asystole ≥ 3 seconds or escape rate < 40 bpm or escape rhythm below AV node

(4)  Associated with atrial fibrillation and pauses ≥ 5 seconds

c.  Symptomatic second-degree AV block (regardless of type)

d.  Unexplained syncope and chronic bifascicular block (especially if accompanied by an HV interval ≥ 70 ms on EPS)

Alternative Diagnosis: PE

Textbook Presentation

PE is covered extensively in Chapter 15, Dyspnea. This discussion will focus on patients who experience syncope due to PE.

The classic presentation of a patient with PE and syncope is an older patient with risk factors for venous thromboembolic disease with the sudden onset of chest pain, dyspnea, and sudden loss of consciousness.

Disease Highlights

A.  PE is a more common cause of syncope than commonly appreciated. PE was diagnosed in 17% of patients admitted for their first episode of syncope who did not have an obvious cause (eg, dehydration, vasovagal).

B.  Syncope complicates PE in 9–24% of patients.

C.  Syncope is secondary to massive embolization (involving > 50% of the pulmonary vascular bed), critically limiting blood return to the LV, reducing cardiac output, causing hypotension and syncope.

Evidence-Based Diagnosis

A.  Echocardiography reveals RV dysfunction in 88–94% of patients with PE and syncope (due to the extent of embolization required for syncope to occur).

B.  Patients with PE and syncope who survive to arrive at the hospital have often stabilized (probably due to clot fragmentation and improved LV return), may be hemodynamically stable and relatively asymptomatic.

C.  25% of patients with PE and syncope had no other symptoms or signs suggestive of PE (increasing the likelihood of missed diagnoses).

PE should be considered in patients with syncope of unknown origin, even in the absence of other clinical symptoms and signs.

D.  Patients with PE and syncope may have typical risk factors, associated symptoms (eg, chest pain or dyspnea) and signs suggesting PE.

E.  Findings suggestive of PE in such patients with syncope include tachypnea (LR+, 6.4) and unilateral leg swelling (LR+, 8.9).

F.  Other findings that might suggest PE include

1.  Persistent hypotension

2.  Hypoxia (PaO2 < 60 mm Hg)

3.  ECG findings (S1Q3T3 pattern, right axis deviation, or right BBB)

4.  Radiographic findings (an unexplained pleural effusion or infiltrate suggestive of pulmonary infarction)

5.  Echocardiographic findings of right atrial or right ventricular enlargement

G.  D-dimer assays and CT angiogram are the most commonly used tests to evaluate patients with possible PE.

Treatment

See Chapter 15.

CASE RESOLUTION

After 24 hours, Mr. C is feeling well. He is anxious to go home. The telemetry reveals normal sinus rhythm without evidence of intermittent AV block or VT. Stress testing is performed and shows evidence of prior MI but no acute ischemia. A D-dimer level is normal.

The sensitivity of telemetry is inadequate to exclude life-threatening arrhythmias such as VT. Furthermore, none of the alternative diagnoses (such as PE, SSS, or AV heart block) are suggested by the history, physical exams, or laboratory test results. After careful discussion with Mr. C, you order an EPS.

The EPS demonstrates inducible sustained VT, placing the patient at high risk for spontaneous lethal ventricular arrhythmias. An ICD is placed. At follow-up 12 months later, Mr. C is doing well and has no subsequent syncopal events. His ICD has delivered 2 shocks.

CHIEF COMPLAINT

PATIENT

Mrs. S is a 60-year-old woman who arrives at the emergency department via ambulance after an episode of transient loss of consciousness. The patient reports that she was eating dinner, and the next thing she knew she was in the emergency department. Mr. S reports that he found his wife lying on the floor next to the dining room table when he came home. At that time, Mrs. S was conscious but lethargic. The food and plate were scattered on the floor. There was no evidence of incontinence. On physical exam, her vital signs are normal. HEENT exam reveals a contusion over the right eye and bruising along the right half of her tongue. Cardiac and pulmonary exams are normal. Abdominal exam is unremarkable. Stool is guaiac negative. Neurologic exam is nonfocal.

At this point, what is the leading hypothesis, what are the active alternatives, and is there a must not miss diagnosis? Given this differential diagnosis, what tests should be ordered?

RANKING THE DIFFERENTIAL DIAGNOSIS

As noted previously, the first pivotal step in the evaluation of patients with transient loss of consciousness is to determine if the loss of consciousness was due to syncope or some nonsyncopal cause (Figure 31-1). Furthermore, the cardinal features of syncope are an abrupt onset, brief duration, and a spontaneous unaided recovered. The remarkable feature of Mrs. S’s history is the prolonged period of lethargy and confusion that persisted until she reached the emergency department. This is a pivotal clue that suggests a nonsyncopal etiology for her loss of consciousness. Diagnostic considerations include seizure, hypoglycemia, intoxication, or another neurologic event (eg, an ischemic event involving the posterior circulation or trauma). The patient’s bruised tongue is a diagnostic fingerprint that markedly increases the likelihood of a seizure. Hypoglycemia-induced syncope is usually preceded by either confusion or sympathetic stimulation producing tremulousness, nervousness, or diaphoresis and occurs almost exclusively in diabetic patients taking insulin, sulfonylureas, or thiazolidinediones. Table 31-6 lists the differential diagnosis.

Table 31-6. Diagnostic hypotheses for Mrs. S.

Patients with syncope should be asked, “What was the next thing you remember?” Patients who do not remember the ambulance ride or suffer a period of amnesia following the event (> 5 minutes) should be evaluated for seizures.

PATIENT

The patient reports no prior history of epilepsy, CNS tumor, or stroke (which can increase the likelihood of seizures). She has no history of diabetes and is not taking any medications. She has no history of cerebrovascular disease, hypertension, or atrial fibrillation. She denies having any focal weakness, dysarthria, diplopia, or difficulty walking. She has no history of head trauma. She denies any recent alcohol or drug use.

Is the clinical information sufficient to make a diagnosis? If not, what other information do you need?

Leading Hypothesis: Seizures

Textbook Presentation

Generalized seizures classically present with tonic-clonic activity, loss of consciousness and postural tone, incontinence, and a prolonged postictal period of lethargy. The purpose of this review is to focus on features that help distinguish seizures from syncope.

Disease Highlights

A.  3% of the US population suffers a seizure in their lifetime

B.  Seizures are the cause of transient loss of consciousness (that may mimic syncope) in 1–7% of patients.

C.  Etiology of seizure and prevalence in patients over age 60

1.  Idiopathic, 35%

2.  Ischemic, 49%

3.  CNS tumor, 11% (about 1/3 primary, 2/3 metastatic)

4.  CNS trauma, 3%

5.  CNS infection, 2%

6.  Metabolic disturbances

a.  Hypoglycemia and hyperglycemia (marked)

b.  Hypoxia

c.  Hyponatremia

d.  Hypocalcemia

e.  Uremia

7.  Medications

a.  Numerous medications have been implicated.

b.  Some commonly used medications that cause seizures (albeit rarely) include cyclosporine, fentanyl, meperidine, lidocaine, phenothiazines, quinolones, theophylline, tricyclic antidepressants, and bupropion.

8.  Illicit drugs ie, MDMA (Ecstasy), cocaine

9.  Withdrawal states (ie, alcohol, baclofen, benzodiazepines, and opioids)

Evidence-Based Diagnosis

A.  FP: Tongue laceration, head turning, and unusual posturing are the most specific clinical features and substantially increase the likelihood of seizure (specificity, 97%, LR+, 12–15) (see Table 31-7).

Table 31-7. Sensitivity, specificity, and LRs for seizures.

B.  Most patients with generalized seizures have postictal confusion. The absence of a postictal period makes generalized seizures unlikely (sensitivity, 94%, LR−, 0.09).

C.  Certain symptoms are unusual in patients with seizures and their presence reduces the likelihood of seizure.

1.  Diaphoresis preceding spell, LR 0.17

2.  Chest pain preceding spell, LR 0.15

3.  Palpitations, LR 0.12

4.  Dyspnea prior to spell, LR 0.08

5.  CAD, LR 0.08

6.  Syncope with prolonged standing, LR 0.05

D.  Convulsive syncope

1.  Limb jerking is not specific for seizures.

2.  15–90% of patients with syncope not related to seizures experience limb jerking, a phenomenon referred to as convulsive syncope. Limb jerking due to syncope is associated with myoclonic jerks, which should be distinguished from tonic-clonic activity.

a.  Myoclonic jerks tend to be arrhythmic and asymmetric, whereas the opposite is true of tonic-clonic activity.

b.  Myoclonic jerks tend to be briefer (average of 6.6 seconds) than tonic-clonic activity seen in seizures (≈ 1 minute)

c.  Myoclonic jerks never precede collapse, whereas tonic-clonic activity may precede collapse.

3.  Finally, unlike generalized seizures, which are usually associated with a significant postictal period, convulsive syncope is not associated with a significant postictal period (< 1 minute).

4.  Patients who appear to have refractory “seizure disorders” and nonspecific abnormalities on electroencephalogram (EEG) should undergo a cardiac evaluation to rule out convulsive syncope with myoclonic jerks.

E.  A point score to distinguish seizures from syncope has been developed (Table 31-8). Point scores of ≥ 1 suggest seizures (sensitivity, 94%; specificity, 94%; LR+, 16; LR−, 0.06).

Table 31-8. A point score to distinguish seizures from syncope.1

F.  Evaluation

1.  EEG

a.  Indicated in the evaluation of patients with possible seizures

b.  Sensitivity is low between episodes (35–50%), but increases with sleep deprivation

c.  Specificity 98%

2.  Neuroimaging

a.  37% of adults with new-onset seizures have structural lesions (eg, tumors, strokes) as do 15% of those without focal neurologic findings.

b.  Indicated in all adults with new-onset seizures.

c.  In acute cases, a noncontrast CT is often performed to rule out an intracranial bleed. Follow-up MRI is recommended due to its increased sensitivity for both tumor and stroke.

3.  Tilt-table testing can be used to help distinguish syncope from seizures when the diagnosis is uncertain. Reproduction of symptoms with tilt-table testing associated with hypotension clearly suggests syncope

4.  ILR have also been used in patients with suspected but unproven seizures refractory to medical therapy and documented arrhythmias in 26%.

5.  Sodium, calcium, glucose, BUN, creatinine, and oxygen saturation should be measured.

6.  Lumbar puncture

a.  A lumbar puncture should be considered if CNS infection is suspected (ie, patient is immunocompromised or has fever, meningismus, headache, or persistent confusion).

b.  Elevated intracranial pressure should be excluded prior to a lumbar puncture (usually with neuroimaging) in order to prevent lumbar puncture–induced herniation.

c.  Platelet count, prothrombin time, and partial thromboplastin time should be checked prior to lumbar puncture. (Thrombocytopenia and coagulopathies increase the risk of bleeding at the lumbar puncture site and subsequent spinal cord compression secondary to hemorrhage.)

7.  Toxicology screen should be ordered if illicit drug use is suspected.

Treatment

Anticonvulsant therapy is complex and evolves rapidly (see neurology texts).

An EEG is ordered to evaluate the patient for possible seizures.

The patient’s bruised tongue and postictal period strongly suggest seizures despite the lack of a previously known seizure disorder or witnesses to the event. You also wonder if an acute stroke is likely and if additional imaging of the extracranial or intracranial vessel is warranted.

Alternative Diagnosis: Cerebrovascular Disease & Syncope

Although physicians commonly consider carotid artery obstruction in the differential diagnosis of patients with syncope, syncope requires transient global cerebral hypoperfusion and unilateral obstruction of the carotid will not result in syncope. Therefore, evaluation of the anterior circulation is not indicated in the patient with syncope. On the other hand, obstruction of the posterior circulation may cause transient loss of consciousness by causing ischemia in the reticular activating system. This may occur in the subclavian steal syndrome, vertebrobasilar insufficiency, and basilar artery occlusion. These disorders are almost invariably associated with neurologic signs or symptoms and should be considered whenever patients have syncope and other symptoms referable to the brainstem (ie, diplopia, vertigo, ataxia, and weakness) (see Chapter 14, Dizziness). Finally, patients in whom subarachnoid hemorrhage develops can present with syncope. Such patients inevitably also complain of severe headache or confusion. Evaluation includes emergent noncontrast head CT scan.

MAKING A DIAGNOSIS

The patient’s EEG revealed intermittent right temporal spike and wave pattern.

The EEG confirms new-onset seizures. Since structural lesions are common in adults with new-onset seizures, neuroimaging is required.

CASE RESOLUTION

An MRI scan revealed a solitary right temporal lobe mass. Subsequent biopsy demonstrated a glioblastoma multiforme. The patient underwent surgical resection and was treated with anticonvulsant therapy. She died approximately 6 months later.

CHIEF COMPLAINT

PATIENT

Mrs. P is a 42-year-old woman who arrives at the emergency department via ambulance with abdominal pain and syncope. She was in her usual state of health until the morning of admission when increasing left lower quadrant abdominal pain developed. The pain increased in intensity and became quite severe. Upon standing, she lost consciousness and collapsed to the floor. She recovered quickly and was helped to a chair by her husband. When she stood several minutes later, she briefly lost consciousness again. The patient reports that her abdominal pain is much better. She has no chest pain or dyspnea. Her vital signs are BP, 105/60 mm Hg; pulse, 85 bpm; temperature, 37.0°C; and RR, 18 breaths per minute. Her cardiac and pulmonary exams are normal, and abdominal exam reveals mild left lower quadrant tenderness. Her ECG is normal and her HCT is normal at 36.0%.

At this point, what is the leading hypothesis, what are the active alternatives, and is there a must not miss diagnosis? Given this differential diagnosis, what tests should be ordered?

RANKING THE DIFFERENTIAL DIAGNOSIS

As noted in Figure 31-1 the first step ascertains whether Mrs. P suffered from syncope or some other transient loss of consciousness. The history of abrupt onset and rapid recovery without intervention strongly suggests syncope. The next step considers whether this is likely due to reflex syncope, orthostatic syncope, or cardiac syncope (Figure 31-2). Several features of Mrs. P’s syncope are noteworthy. First, her syncope occurred in association with abdominal pain raising the possibility of vasovagal syncope. Second, she had 2 episodes of syncope upon standing. This pivotal clue raises the possibility of orthostatic syncope from either dehydration, hemorrhage, medications, or autonomic dysfunction. Finally, cardiac syncope should be considered in all patients with syncope. Fortunately, Mrs. P has no prior history of heart disease that would increase the likelihood of cardiac syncope. Additionally, she has no suggestive symptoms (syncope with chest pain, syncope with exertion, syncope while sitting or supine, palpitations, or dyspnea), or signs (significant murmur, gallop, or JVD) to suggest cardiac syncope. Her ECG is also normal. The combination of the lack of underlying heart disease or suggestive symptoms of cardiac syncope, coupled with recurrent syncope immediately after standing makes orthostatic syncope likely and cardiac syncope unlikely. Table 31-9 lists the differential diagnosis.

Table 31-9. Diagnostic hypotheses for Mrs. P.

Further history reveals that Mrs. P is not taking any medications. Your initial assessment is a neurocardiogenic syncope secondary to transient abdominal pain.

As discussed in the first case presentation, vasovagal syncope is often precipitated by pain, is brief, and is followed by a rapid restoration of consciousness. Many of Mrs. P’s features are consistent with this diagnosis. However, both episodes of syncope occurred immediately after standing providing a clue that her syncope was in fact orthostatic. In addition, although her abdominal pain is improved, it is still unexplained. You elect to check her orthostatic BP and pulse, which is a key component of the evaluation of patients with syncope (Figure 31-2).

Mrs. P’s BP while supine was 105/60 mm Hg with a pulse of 85 bpm, which changed when sitting to BP of 95/50 mm Hg with a pulse of 90 bpm. Upon standing her BP fell to 60/0, her pulse was 140 bpm, and she lost consciousness. She was quickly laid down and again rapidly regained consciousness.

Orthostatic measurement of BP and pulse are critical. Life-threatening hypovolemia may be overlooked if the BP and pulse are not measured while the patient is standing.

Mrs. P’s profound drop in BP upon standing and recurrent syncope is a key pivotal clue and clearly indicate that her syncope is due to orthostatic hypotension. This is not consistent with reflex syncope. You revise the leading hypothesis to syncope due to orthostatic hypotension.

Is the clinical information sufficient to make a diagnosis? If not, what other information do you need?

Leading Hypothesis: Orthostatic Hypotension

Textbook Presentation

Orthostatic hypotension usually becomes symptomatic when patients stand. Patients typically note that they passed out almost immediately after standing from a chair or arising from bed. Other symptoms include near syncope, weakness, visual blurring, weakness, or leg buckling.

Disease Highlights

A.  Orthostatic hypotension occurs in 20% of patients over age 75 and accounts for 12–30% of patients with syncope.

B.  Classic orthostatic hypotension occurs within 3 minutes of standing. Orthostatic hypotension may also be delayed for > 3–10 minutes or develop rapidly, be transient, and difficult to detect with standard sphygmomanometry.

1.  Transient orthostatic hypotension is defined as a drop in systolic BP > 40 mm Hg or diastolic BP > 20 mm Hg within 15 seconds of standing.

2.  Due to its rapid improvement, this cannot be detected without beat-to-beat BP measurements, but patients may be symptomatic nonetheless.

3.  One study documented that 27% of patients with orthostatic dizziness but without demonstrable orthostatic hypotension had a history of syncope, suggesting that many of these patients had transient or intermittent orthostatic hypotension that was undetected. Another study documented that this accounted for 8% of syncope cases among young adults.

C.  Etiology

1.  Hypovolemia

a.  Dehydration

(1)  Decreased oral intake

(2)  GI losses (vomiting, diarrhea)

(3)  Urinary losses

(a)  Uncontrolled diabetes mellitus

(b)  Salt-losing nephropathy

(c)  Adrenal insufficiency

b.  Hemorrhage

(1)  GI

(2)  Ruptured abdominal aortic aneurysm

(3)  Ruptured spleen

(4)  Ruptured ectopic pregnancy

Intra-abdominal and retroperitoneal bleeding are uncommon causes of orthostatic syncope but can be life-threatening and occult! They should be considered in patients with marked orthostatic hypotension of unclear cause particularly if there is abdominal or back pain.

c.  Overdialysis

d.  Postprandial hypotension, particularly common in the elderly and worse with large carbohydrate meals or alcohol ingestion. Splanchnic pooling decreases venous return.

e.  Hot environments (hot tubs, baths, saunas)

2.  Medications

a.  Alpha- and beta-blockers

b.  Diuretics

c.  Vasodilators (ie, nitrates, calcium channel blockers, hydralazine)

d.  ACE inhibitors and angiotensin receptor blockers

e.  Tricyclic antidepressants

f.  Antipsychotic and anti-parkinsonism drugs

g.  Sildenafil and other phosphodiesterase inhibitors particularly when combined with nitrates

h.  Alcohol, sedative hypnotics, and opioids

3.  Autonomic insufficiency: Many (but not all) such patients demonstrate a fall in BP upon standing without a concomitant increase in pulse.

a.  Central neurologic disorders (eg, Parkinson disease, multisystem atrophy, pure autonomic failure, multiple sclerosis, and numerous others)

b.  Peripheral neurologic disorders: Diabetes mellitus, vitamin B12 deficiency, uremia, Lyme disease, syphilis, HIV and other causes of autonomic neuropathies

c.  Prolonged bed rest

Evidence-Based Diagnosis

A.  Definition of orthostatic hypotension

1.  ≥ 20 mm Hg decrease in systolic BP within 3 minutes of standing; a decrease in systolic BP of ≥ 30 mm Hg may be a more appropriate criterion in patients with hypertension

2.  ≥ 10 mm Hg decrease in diastolic BP within 3 minutes of standing

3.  Or > 30 bpm increase in pulse within 3 minutes of standing

4.  The European Society of Cardiology also includes a fall in systolic BP to < 90 mm Hg.

B.  20% of the elderly have postural hypotension. Therefore, the presence of orthostatic hypotension alone does not confirm that syncope was secondary to orthostatic hypotension. Syncope from orthostatic hypotension is confirmed in patients with both orthostatic hypotension and syncope or presyncope on standing.

C.  Orthostatic measurements are far more sensitive than isolated seated or supine BP measurements, which can miss profound orthostatic hypotension even in patients with syncope due to life-threatening hemorrhage or marked dehydration. Studies on the impact of moderate volume phlebotomy (450–630 mL) or large volume phlebotomy (630–1150 mL) reveal the following:

1.  Simple supine measurements of BP and pulse were not sensitive to even large blood loss (sensitivity, 12–33%)

2.  An increase in pulse of > 30 bpm with standing is both highly sensitive for large volume (1–2 units) blood loss (97%) and highly specific (98%, LR+, 48) (Table 31-10). The sensitivity falls dramatically if the patient sits instead of stands (39–78%).

Table 31-10. Accuracy of physical exam for large blood loss (1.2–2.2 units).

3.  The sensitivity of orthostatic measurements is greatest if the supine and standing BPs are compared. If the supine BP is not measured, 67% of orthostatic patients may not be identified.

4.  Any abnormal finding on orthostatic maneuvers strongly suggested volume loss (specificity, 94–98%; LR+, 3.0–48).

5.  Patients should stand for 1 minute before the measurement of the upright BP.

6.  No measure was very sensitive for moderate blood loss (0–27%).

Orthostatic vital signs are a key component of the physical exam in patients with syncope or near syncope

7.  Profound blood loss may occasionally paradoxically produce bradycardia. (The reduction in end-systolic volume may trigger the neurally mediated reflex.)

8.  A CBC may be useful but it is critical to appreciate that the initial HCT may not accurately reflect the severity of acute hemorrhage. The HCT may be normal prior to rehydration and the fall in HCT may take 24–72 hours.

D.  Tilt-table testing can be useful to diagnose delayed orthostatic hypotension.

F.  A history of nausea, vomiting, diarrhea, or inadequate intake can suggest volume depletion that is supported by elevations in BUN, creatinine and/or low urinary Na+, FENa+ or FEurea.

Treatment

A.  Acute blood loss: Blood transfusion is appropriate in the orthostatic patient with acute blood loss as well as identification and treatment of the underlying disorder.

B.  Dehydration (diarrhea, vomiting, or decreased oral intake)

1.  Patients able to tolerate oral intake: oral rehydration preferably with sodium-rich beverages

2.  Patients unable to tolerate oral intake: IV hydration

a.  Normal saline is preferred.

b.  Usually 500 mL to 1 L boluses are given over 1 hour.

c.  Smaller boluses may be given to fragile patients (ie, small elderly women or those with a history of kidney disease or HF).

d.  Repeat orthostatic BP measurements are made following each bolus as well as a lung and cardiac exam to ensure the patient has not received excessive fluid.

e.  Bolus therapy should be continued until orthostatic hypotension resolves.

C.  Chronic orthostatic hypotension

1.  Hydration (water, soup, or sports drinks)

2.  Discontinue or decrease offending medications (antihypertensives, especially beta-blockers, thiazides, and alpha-blockers; nitrates; tricyclic antidepressants; phenothiazines).

3.  Counterpressure maneuvers can be tried. (See vasovagal syncope above.)

4.  Patients are advised to arise slowly (sitting on the side of the bed prior to standing), avoid large meals and excessive heat, and use waist high support hose.

5.  Alpha-agonists (ie, midodrine) have also been used successfully. Side effects include urinary retention, hypertension, and worsening HF.

6.  Fludrocortisone has also been used successfully.

MAKING A DIAGNOSIS

Mrs. P reports that she has not suffered from any diarrhea or vomiting and has ingested normal amounts of fluid. She denies any hematemesis, melena, or bright red blood per rectum.

It is important to remember that Mrs. P presented with syncope and abdominal pain. Although the pain has improved, it has not resolved; it may provide an important clue to the underlying etiology. Given the profound orthostatic hypotension and the lack of external blood or volume loss, or incriminating medication, internal bleeding must be considered as a source of her abdominal pain and syncope. In the differential diagnosis you consider splenic rupture, ruptured abdominal aortic aneurysm, and ruptured ectopic pregnancy. The lack of trauma argues against splenic rupture and the patient’s age and gender are atypical for abdominal aortic aneurysm. You wonder if in fact she has suffered from a ruptured ectopic pregnancy.

It is important to remember the patient’s chief complaint because it usually holds the most important clue to the diagnosis.

CASE RESOLUTION

Mrs. P reports that she missed her last menstrual period. An abdominal ultrasound is performed and reveals 750 mL of fluid (presumed to be blood) in the pelvis. A urine pregnancy test is positive.

Although the final diagnosis of ectopic pregnancy was not considered initially, a careful clinical exam confirmed orthostatic syncope. Once that pivotal clue was discovered, the differential diagnosis could be narrowed and the underlying cause determined. It is instructive to note that her initial HCT was normal because the remaining intravascular blood had not yet been diluted by any oral or IV fluids.

Initial HCT measurements will not accurately reflect the magnitude of blood loss in a patient with recent hemorrhage.

Mrs. P had 2 large bore IVs placed and was typed and crossed for RBC transfusions. CBC, prothrombin time, partial thromboplastin time, and platelet counts were measured and a 1 L bolus of normal saline was given while waiting for the packed RBCs. After volume and blood resuscitation, she underwent surgical exploration and removal of her ruptured fallopian tube.

REVIEW OF OTHER IMPORTANT DISEASES

Aortic Stenosis

Textbook Presentation

Aortic stenosis is usually diagnosed incidentally during routine exam rather than due to symptoms. Typically, aortic stenosis produces a loud crescendo-decrescendo systolic murmur at the right second intercostal space, which may radiate to the neck. When aortic stenosis becomes severe, patients may have any of the 3 cardinal symptoms: syncope, angina, or HF (dyspnea, typically with exertion).

Disease Highlights

A.  Thickening and calcification of valve leaflets results in progressive obstruction to blood flow.

B.  LVH develops to compensate for the obstruction.

C.  Pathophysiology of the cardinal symptoms is shown in Figure 31-9.

Figure 31-9. Pathophysiology of symptoms in aortic stenosis.

D.  Prevalence is 3% in patients ≥ 75 years old.

E.  Etiology:

1.  Most commonly due to calcification of a trileaflet aortic valve

2.  Accelerated in patients with congenital bicuspid valve

a.  1–2% of the population is born with a congenital bicuspid valve.

b.  Severe aortic stenosis develops in 66% of patients and at an earlier age than in patients with tricuspid valves.

c.  Aortic root structure is usually also abnormal and often associated with progressive dilation of the aortic root that may require repair to prevent rupture or dissection.

3.  Rheumatic heart disease

F.  Severe aortic stenosis is characterized by a valve area < 1 cm, associated with an increased velocity across the aortic valve (≥ 4 m/s) and mean aortic valve gradient > 40 mm Hg. Gradients and velocities may be lower in patients with a low cardiac output.

G.  Prognosis: Mortality increases markedly when symptoms develop (HF, angina, or syncope). The most common symptoms are decreased exercise tolerance and dyspnea on exertion. Mortality for symptomatic patients if they do not have valve replacement:

1.  Aortic stenosis and angina: 50% 5-year mortality

2.  Aortic stenosis and syncope: 50% 3-year mortality

3.  Aortic stenosis and dyspnea: 50% 2-year mortality

H.  Other late manifestations:

1.  Atrial fibrillation may develop. This is often poorly tolerated because the LV is noncompliant and dependent on atrial contraction for filling.

2.  An increased bleeding tendency secondary to disruption of large von Willebrand multimers by the abnormal aortic valve.

Evidence-Based Diagnosis

A.  History and physical exam: Most studies demonstrate only a fair reproducibility between examiners.

1.  Findings that help suggest aortic stenosis

a.  Effort syncope in patients with a systolic murmur (LR+, 1.3–∞ LR−, 0.76)

b.  Slow carotid upstroke (sensitivity, 15–42%; specificity, 95–100%; LR+, 9.2–∞)

c.  Murmur radiating to right carotid (sensitivity, 71–73%; specificity, 90%; LR+, 7.5)

2.  Findings that help rule out aortic stenosis

a.  Absence of any murmur (LR−, 0.0)

b.  Absence of murmur below right clavicular head (LR−, 0.1)

3.  Murmurs may be less intense in patients with superimposed HF.

B.  Echocardiogram

1.  The initial test of choice to assess for aortic stenosis

2.  Recommended for patients with a systolic murmur ≥ grade III/VI

3.  Criteria for severe aortic stenosis is a flow rate > 4.0 m/s corresponding to a valve area < 1.0 cm2 and a gradient of ≥ 40 mm Hg.

4.  Patients with HF or a small hypertrophied LV may have lower flow rates across the AV, which may underestimate the severity of aortic stenosis.

5.  Surveillance echocardiography is recommended to monitor progression (every 6 months for severe aortic stenosis, and every year for mild to moderate aortic stenosis associated with significant calcification). Echocardiography is also recommended for any change in symptoms or signs and during pregnancy.

6.  Patients with bicuspid aortic valves and aortic stenosis or dilation of the aortic root (> 40 mm) should have annual echocardiography. In patients without stenosis or dilatation, echocardiography is recommended every 2 years.

C.  Stress testing

1.  Should not be performed in patients with severe aortic stenosis and an ejection fraction < 50% or are symptomatic

2.  May be reasonable in asymptomatic patients with severe aortic stenosis and a normal ejection fraction to confirm the absence of symptoms.

Treatment

A.  Since the prognosis changes markedly in symptomatic patients, patients with severe asymptomatic aortic stenosis should be reevaluated every 6 months and queried about HF symptoms, angina, or syncope. Patients should also be instructed to report new symptoms.

B.  Symptomatic patients should undergo mechanical correction, not medical therapy.

C.  While awaiting surgery, patients with HF symptoms may be treated cautiously with diuretics, and ACE inhibitors

D.  Mechanical correction:

1.  Valve replacement is guided almost exclusively by symptoms.

2.  Definite indications for valve replacement

a.  Severe aortic stenosis in symptomatic patients

b.  Severe aortic stenosis with symptoms on exercise testing

c.  Severe aortic stenosis in asymptomatic patients undergoing other cardiac surgery (eg, coronary artery bypass grafting).

d.  Severe aortic stenosis in asymptomatic patients with ejection fraction < 50%.

3.  The 2014 AHA guidelines list several other “reasonable” indications for aortic valve replacement including very severe asymptomatic aortic stenosis (velocity > 5 m/s or mean gradient > 60 mm Hg) in low-risk patients.

4.  Standard preoperative evaluation includes angiography in many patients to determine whether the patient needs concomitant coronary artery bypass surgery. This includes patients with symptoms of CAD, CAD risk factors (including men aged ≥ 40 years, postmenopausal women,) or decreased ejection fraction.

5.  Three options for valve replacement are commonly used: (1) surgically placed mechanical valves, (2) surgically placed bioprosthetic valves, and (3) catheter-inserted bioprosthetic valves (transcatheter aortic valve replacement [TAVR]).

6.  Precise indications continue to evolve. Pertinent pros and cons of the options include:

a.  TAVR does not require open heart surgery and is a reasonable alternative to surgical aortic valve replacement for symptomatic patients with severe aortic regurgitation when the surgical risk is intermediate to high.

(1)  As the name implies, the major advantage of these valves is that they do not require open heart surgery but are placed using a catheter, most commonly deployed through the femoral artery.

(2)  The longevity of these bioprosthetic TAVR valves is not yet clear.

(3)  Perioperative MI, early major bleeding, acute kidney injury, and new-onset atrial fibrillation were less common with TAVR than surgical aortic valve replacement, whereas early vascular complications, need for pacemaker implantation, and perivalvular leaks were more common with TAVR.

(4)  These valves require dual antiplatelet therapy for 3–6 months and then lifelong aspirin therapy.

b.  Surgically placed mechanical valves

(1)  Have greater durability and a significantly lower rate of failure and need for replacement, which is particularly important in patients younger than age 55

(2)  However, they are associated with an increased risk of thromboembolism and require lifelong anticoagulation with vitamin K antagonists.

(3)  Direct-acting oral anticoagulants are not recommended.

(4)  In addition to warfarin, aspirin is recommended at 75–100 mg/day.

c.  Surgically placed bioprosthetic valves

(1)  Require only brief anticoagulation with vitamin K antagonists (for the first 3–6 months following placement)

(2)  However, have a limited longevity and are typically reserved for patients over age 65 (whose life expectancy makes the need for a second replacement unlikely).

(3)  Lifelong aspirin 75–100 mg/day is recommended.

d.  Balloon valvotomy is a less commonly used option.

(1)  Can be used as a palliative procedure but is associated with a high complication rate (> 10%) and provides only temporary relief (6–12 months)

(2)  Reserved for palliation in patients with other serious (or lethal) comorbidities and as a bridge to TAVR or aortic valve replacement in patients who are hemodynamically unstable or require urgent major noncardiac surgery

E.  Vigorous exercise should be discouraged and vasodilators (hydralazine, nitroglycerin, and nifedipine) avoided or used with caution in patients with moderate to severe aortic stenosis.

Situational Syncope

A variant of reflex syncope, situational syncope occurs during or immediately after micturition, defecation, swallowing, or coughing, which increase vagal tone.

Carotid Sinus Syndrome

Textbook Presentation

Carotid sinus syndrome (CSS) is another variant of reflex syncope. Patients typically complain of syncope or falls that may be precipitated when pressure is inadvertently applied to the carotid (eg, head turning, buttoning collar, shaving, or cervical motion) or occur spontaneously.

Disease Highlights

A.  Carotid sinus hypersensitivity (CSH) represents a syndrome of increased carotid sensitivity with resultant hypotension due to bradycardia, vasodilation, or both.

B.  Increasingly common in the elderly. Accounts for 8–15% of syncopal events in patients older than 40 years with syncope of unclear etiology.

C.  Mean age 77 and unusual in patients younger than 40 years.

D.  15–56% of affected patients complain of falls but deny syncope.

1.  May be due to retrograde amnesia or alternatively, hypotension that is insufficient to maintain an upright posture but sufficient to avoid frank loss of consciousness.

2.  CSS present in 19–27% of patients with unexplained falls but 0% in patients with accidental falls.

Consider CSS in elderly patients with unexplained falls.

Evidence-Based Diagnosis

A.  Carotid sinus massage is used in the diagnosis of CSS.

1.  Carotid sinus massage is applied (unilaterally) for 5–10 seconds during continuous ECG and BP monitoring. Carotid sinus massage needs to be performed on each side separated in time by ≥ 1 minute and in both the supine and upright position.

2.  The inhibitory response is unilateral in 81% of patients, suggesting the need to evaluate both carotids.

3.  30–50% of patients have findings only when upright.

4.  Carotid sinus massage has been complicated by transient or permanent neurologic symptoms in 0.24%. It is contraindicated in patients with carotid bruits, known carotid stenosis > 70%, recent cerebrovascular accident, transient ischemic attack, or MI within the last 3 months.

B.  CSH should be distinguished from CSS.

1.  CSH is defined as a ≥ 3-second pause or decrease in systolic BP (≥ 50 mm Hg) accompanying carotid sinus massage.

2.  CSS is diagnosed in patients with a ≥ 3-second pause or decrease in systolic BP (≥ 50 mm Hg) accompanying carotid sinus massage and syncope of unknown origin, compatible with a reflex mechanism.

3.  Some authors also recommend diagnosing CSS in patients with symptoms during carotid sinus massage and an absolute systolic BP ≤ 85 mm Hg to increase sensitivity (from 93% to 100%).

4.  CSH without concurrent syncope is nonspecific and not diagnostic as it occurs in 35% of asymptomatic elderly patients without prior falls, dizziness, or syncope.

5.  CSS appears to be more specific. It affects only 5% of elderly patients without prior falls, dizziness, or syncope. Additionally, although this elderly population with CSS often has other potential explanations for syncope (74%), alternative diagnoses were made in only 8% of patients in whom CSS was diagnosed during follow-up.

C.  47% of patients report symptoms precipitated by looking upward.

Treatment

The American Heart Association and European Society of Cardiology concluded that pacemakers are reasonable and should be considered in syncopal patients with cardioinhibitory CSS with documented asystole of ≥ 3 sec with carotid sinus massage in whom they have been demonstrated to reduce the incidence of subsequent syncope and falls (from 38% to 9%).

Wolff-Parkinson-White (WPW) Syndrome

Textbook Presentation

WPW syndrome may be asymptomatic or present with palpitations, dyspnea, near syncope, syncope, or sudden death. In some patients, the diagnosis may only be made after typical ECG findings are discovered on an ECG performed for some other reason.

Disease Highlights

A.  A congenital disorder in which an accessory bundle directly connects the atria and ventricular muscle bypassing the AV node.

B.  While many patients with the WPW syndrome remain asymptomatic, a variety of life-threatening tachyarrhythmias may occur:

1.  In orthodromic tachycardia, an impulse spreads down the rapidly conducting His-Purkinje system and then back up the accessory pathway in a retrograde fashion (Figure 31-10) to the atria. Because the ventricle is activated by the His-Purkinje system, the QRS complex is narrow.

Figure 31-10. Orthodromic tachycardia in patients with the Wolff-Parkinson-White syndrome. (Reproduced with permission from McPhee SJ: Pathophysiology of Disease, 5th ed. New York, NY: McGraw-Hill Education; 2006.)

2.  In antidromic tachycardia, the reentrant loop runs in the opposite direction down the accessory pathway. Since ventricular depolarization starts at the insertion site of the accessory pathway (and not via the rapidly conducting His-Purkinje system), it depolarizes from cell to cell resulting in a wide complex tachycardia.

3.  Finally, atrial fibrillation or flutter can develop. The accessory pathway conducts impulses directly into the ventricles (bypassing the AV node), allowing very rapid ventricular depolarization and putting patients at risk for ventricular fibrillation and sudden death.

C.  Syncope in patients with WPW syndrome is associated with a greater frequency (25%) of rapid, life-threatening, conduction over the accessory pathway.

Evidence-Based Diagnosis

A.  Baseline ECG abnormalities during normal sinus rhythm may reveal a combination of a short PR interval and a delta wave.

1.  Short PR interval (< 0.12 s)

a.  In healthy persons, the normal PR interval is produced by a built-in delay at the AV node (designed to allow atrial emptying prior to ventricular systole).

b.  In WPW syndrome, the accessory pathway bypasses the AV node and initiates ventricular depolarization without such a delay; this results in a shortened PR interval in 75% of patients (Figure 31-11).

Figure 31-11. Electrocardiographic features of the Wolff-Parkinson-White syndrome. (Reproduced with permission from Fuster V: Hurst’s The Heart, 12th ed. New York, NY: McGraw-Hill Education; 2008.)

2.  Delta wave

a.  In most patients with WPW syndrome, the accessory pathway inserts directly into the ventricular muscle (rather than into the specialized His-Purkinje system).

b.  Ventricular depolarization spreads slowly from cell to cell through gap junctions, rather than rapidly through the specialized His-Purkinje conduction system.

c.  This results in slow ventricular depolarization and the slow initial upstroke of the QRS complex known as the delta wave (Figure 31-11).

d.  Finally, as this ventricular depolarization progresses, the AV node is also processing the supraventricular impulse. Eventually, the impulse passes through the AV node, activates the His-Purkinje system, and causes rapid depolarization. This results in a narrow terminal portion of the QRS complex.

Treatment

A.  Risk stratification

1.  Stress tests: In certain patients, the accessory pathways cannot conduct at rapid heart rates, reducing the patient’s risk of life-threatening arrhythmias. This may be apparent on stress tests if there is an abrupt loss of the delta wave and short PR intervals at rapid heart rates.

2.  EPS can be useful for diagnosis, prognosis, and therapy.

a.  EPS can confirm the presence of a bypass tract.

b.  EPS can measure the conduction characteristics of the bypass tract to determine if it can sustain rapid life-threatening arrhythmias.

c.  Radiofrequency ablation can obliterate the bypass tract in high-risk patients.

d.  An EPS is typically offered to the following patients:

(1)  Symptomatic patients (eg, history of tachycardia, syncope, or sudden death)

(2)  Patients with structural heart disease who are at a higher risk for atrial fibrillation

(3)  In selected asymptomatic patients whose stress test does not demonstrate loss of preexcitation at rapid heart rates

B.  Treatment

1.  Immediate management

a.  A variety of options exist for patients with acute WPW-related arrhythmias, including electrical cardioversion and pharmacotherapy.

b.  The choice of pharmacotherapy depends on the mechanism of the tachycardia.

c.  Consultation is recommended.

2.  Long-term management

a.  Long-term therapeutic options include pharmacologic therapy and radiofrequency catheter ablation of the bypass tract.

b.  Radiofrequency ablation can be performed if the risk of rapid tachycardias is high.

c.  Consultation is recommended.

REFERENCES

Alboni P, Brignole M, Menozzi C et al. Diagnostic value of history in patients with syncope with or without heart disease. J Am Coll Cardiol. 2001;37(7):1921–8.

Anpalahan M, Gibson S. The prevalence of neurally medicated syncope in older patients presenting with unexplained falls. Eur J Intern Med. 2012;23:e48–e52.

Bell WR, Simon TL, DeMets DL. The clinical features of submassive and massive pulmonary emboli. Am J Med. 1977;62(3):355–60.

Birnbaum A, Esses D, Bijur P, Wollowitz A, Gallagher EJ. Failure to validate the San Francisco Syncope Rule in an independent emergency department population. Ann Emerg Med. 2008;52:151–9.

Brignole M, Moya A, de Lange FJ et al. 2018 ESC Guidelines for the diagnosis and management of syncope. Eur Heart J. 2018;39(21):1883–1948.

Calkins H, Shyr Y, Frumin H, Schork A, Morady F. The value of the clinical history in the differentiation of syncope due to ventricular tachycardia, atrioventricular block, and neurocardiogenic Syncope. Am J Med. 1995;98:365–73.

Calvo-Romero JM, Perez-Miranda M, Bureo-Dacal P. Syncope in acute pulmonary embolism. Eur J Emerg Med. 2004;11(4):208–9.

Castelli R, Tarsia P, Tantardini C, Pantaleo G, Guariglia A, Porro F. Syncope in patients with pulmonary embolism: comparison between patients with syncope as the presenting symptom of pulmonary embolism and patients with pulmonary embolism without syncope. Vasc Med. 2003;8(4):257–61.

Colman N, Nahm K, van Dijk JG, Reitsma JB, Wieling W, Kaufmann H. Diagnostic value of history taking in reflex syncope. Clin Auton Res. 2004;14 Suppl 1:37–44.

Colman N, Bakker A, Linzer M, Reitsma JB, Wieling W, Wilde AA. Value of history-taking in syncope patients: in whom to suspect long QT syndrome. Europace. 2009;11:937–43.

Corrado D, Basso C, Pavei A, Michieli P, Schiavon M, Thiene G. Trends in sudden cardiovascular death in young competitive athletes after implementation of a preparticipation screening program. JAMA. 2006;296:1593–1601.

Davies AJ, Steen N, Kenny RA. Carotid sinus hypersensitivity is common in older patients presenting to an accident and emergency department with unexplained falls. Age Ageing. 2001;30:289–93.

Elliott PM, Anastasakis A, Borger MA et al. 2014 ESC Guidelines on diagnosis and management of hypertrophic cardiomyopathy. Eur Heart J. 2014;35:2733–79.

Epstein AE, DiMarco JP, Ellenbogen KA et al. 2012 ACCF/AHA/HRS 2012 focused update incorporated into the ACCF/AHA/HRS 2008 guidelines for device-based therapy of cardiac rhythm abnormalities. J Am Coll Cardiol. 2013;61(3):36–75.

Epstein AE, DiMarco JP, Ellenbogen KA et al. ACC/AHA/HRS 2008 Guidelines for Device-Based Therapy of Cardiac Rhythm Abnormalities: a report of the American College of Cardiology/American Heart Association Task Force on Practice Guidelines (Writing Committee to Revise the ACC/AHA/NASPE 2002 Guideline Update for Implantation of Cardiac Pacemakers and Antiarrhythmia Devices): developed in collaboration with the American Association for Thoracic Surgery and Society of Thoracic Surgeons. Circulation. 2008;117(21):e350–408.

Etchells E, Bell C, Robb K. Does this patient have an abnormal systolic murmur? JAMA. 1997;277(7):564–71.

Etchells E, Glenns V, Shadowitz S, Bell C, Siu S. A bedside clinical prediction rule for detecting moderate or severe aortic stenosis. J Gen Intern Med. 1998;13(10):699–704.

European Heart Rhythm Association; Heart Rhythm Society, Zipes DP, Camm AJ, Borggrefe M et al; American College of Cardiology; American Heart Association Task Force; European Society of Cardiology Committee for Practice Guidelines. ACC/AHA/ESC 2006 guidelines for management of patients with ventricular arrhythmias and the prevention of sudden cardiac death: a report of the American College of Cardiology/American Heart Association Task Force and the European Society of Cardiology Committee for Practice Guidelines (Writing Committee to Develop Guidelines for Management of Patients With Ventricular Arrhythmias and the Prevention of Sudden Cardiac Death). J Am Coll Cardiol. 2006;48(5):e247–346.

Gersh BJ, Maron BJ, Bonow RO et al. 2011 ACCF/AHA Guideline for the Diagnosis and Treatment of Hypertrophic Cardiomyopathy: Executive Summary: A Report of the American College of Cardiology Foundation/American Heart Association Task Force on Practice Guidelines. Circulation. 2011;124:2761–96.

Goldberger ZD, Rho RW, Page L. Approach to the diagnosis and initial management of the stable adult patient with a wide complex tachycardia. Am J Cardiol. 2008;101:1456–66.

Graf D, Schlaepfer J, Gollut E et al. Predictive models of syncope causes in an outpatient clinic. Int J Cardiol. 2008;123:249–56.

Grubb BP. Clinical practice. Neurocardiogenic syncope. N Engl J Med. 2005;352(10):1004–10.

Joint Task Force on the Management of Valvular Heart Disease of the European Society of Cardiology (ESC); European Association for Cardio-Thoracic Surgery (EACTS), Vahanian A, Alfieri O, Andreotti F et al. Guidelines on the management of valvular heart diseases (version 2012). Eur Heart J. 2012;33:2451–96.

Kerr SR, Pearce MS, Brayne C, Davis RJ, Kenny RA. Carotid sinus hypersensitivity in asymptomatic older persons. Implications for diagnosis of syncope and falls. Arch Intern Med. 2006;166:515–20.

Lakdawala NK, Thune JJ, Maron BJ et al. Electrocardiographic features of sarcomere mutation carriers with and without clinically overt hypertrophic cardiomyopathy. Am J Cardiol. 2011;108:1606–13.

Lembo NJ, Dell’Italia LJ, Crawford MH, O’Rourke RA. Bedside diagnosis of systolic murmurs. N Engl J Med. 1988;318(24):1572–8.

Maron BJ, Casey SA, Chan RH, Garberich RF, Rowin EJ, Maron MS. Independent Assessment of the European Society of Cardiology Sudden Death Risk Model for Hypertrophic Cardiomyopathy. Am J Cardiol. 2015;116:757–64.

McDermott D, Quinn JV, Murphy CE. Acute myocardial infarction in patients with syncope. CJEM. 2009;11(2):156–60.

McGee S, Abernethy WB 3rd, Simel DL. The rational clinical examination. Is this patient hypovolemic? JAMA. 1999;281(11):1022–9.

McKeon A, Vaughan C, Delanty N. Seizure versus syncope. Lancet Neurol. 2006;5(2):171–80.

Mitro P, Kirsch P, Valočik G, Murín P. Clinical history in the diagnosis of the cardiac syncope—the predictive scoring system. Pacing Clin Electrophysiol. 2011;34:1480–5.

Moya A, Brignole M, Menozzi C et al; International Study on Syncope of Uncertain Etiology (ISSUE) Investigators. Mechanism of syncope in patients with isolated syncope and in patients with tilt-positive syncope. Circulation. 2001;104:1261–7.

Nishimura RA, Otto CM, Bonow RO et al. 2017 AHA/ACC Focused Update of the 2014 AHA/ACC Guideline for the Management of Patients with Valvular Heart Disease. JACC. 2017;70(2):252–89.

Nishimura RA, Otto CM Bonow RO et al. 2014 AHA/ACC guideline for the management of patients with valvular heart disease: executive summary: a report of the American College of Cardiology/American Heart Association Task Force on Practice Guidelines. Circulation. 2014;63(22):2438–88.

Nishimura RA, Holmes DR Jr. Clinical practice. Hypertrophic obstructive cardiomyopathy. N Engl J Med. 2004;350(13):1320–7.

O’Mahony C, Tome-Esteban M, Lambiase PD et al. A validation study of the 2003 American College of Cardiology/European Society of Cardiology and 2011 American College of Cardiology Foundation/American Heart Association risk stratification and treatment algorithms for sudden cardiac death in patients with hypertrophic cardiomyopathy. Heart. 2013;99:534–41.

Pezawas T, Stix G, Kastner J, Schneider B, Wolzt M, Schmidinger H. Implantable loop recorder in unexplained syncope: classification, mechanism, transient loss of consciousness and role of major depressive disorder in patients with and without structural heart disease. Heart. 2008;94:e17.

Prandoni P, Lensing AW, Prins MH et al. Prevalence of pulmonary embolism among patients hospitalized for syncope. N Engl J Med. 2016;375:1524–31.

Russo AM, Stainback RF, Bailey SR et al. ACCF/HRS/AHA/ASE/HFSA/SCAI/SCCT/SCMR 2013 appropriate use criteria for implantable cardioverter-defibrillators and cardiac resynchronization therapy. Heart Rhythm. 2013;10(4):1–48.

Sheldon R, Rose S, Connolly S, Ritchie D, Koshman ML, Frenneaux M. Diagnostic criteria for vasovagal syncope based on a quantitative history. Eur Heart J. 2006;27(3):344–50.

Sheldon R, Rose S, Ritchie D et al. Historical criteria that distinguish syncope from seizures. J Am Coll Cardiol. 2002;40(1):142–8.

Shen WK, Sheldon RS, Benditt DG et al. 2017 ACC/AHA/HRS Guideline for the evaluation and management of patients with syncope: executive summary: a report of the American College of Cardiology/American Heart Association/Task Force on Clinical Practice Guidelines and the Heart Rhythm Society. Circulation. 2017;136(5):e25–e59.

Solano A, Menozzi C, Maggi R et al. Incidence, diagnostic yield and safety of the implantable loop-recorder to detect the mechanism of syncope in patients with and without structural heart disease. Eur Heart J. 2004;25:1116–9.

Ungar A, Sgobino P et al. Diagnosis of neurally mediated syncope at initial evaluation and with tilt table testing compared with that revealed by prolonged ECG monitoring. An analysis from the Third International Study of Syncope of Uncertain Etiology (ISSUE-3). Heart. 2013;99:1825–31.

Warren J. The effect of venesection and the pooling of blood in the extremities on the atrial pressure and cardiac output in normal subjects with observations on acute circulatory collapse in three instances. J Clin Invest. 1945 May;24(3):337–44.